You are on page 1of 84

Zackary Gibbons- Crim.Pro.

Adjudication

I.

Regulating Charging Discretion of Prosecutors A. Case Assignment 1. Vertical a. Complaint once filed will be assigned to a specified case flow resulting in its eventual presentation in particular courtroom or before particular judge b. Single AP or team of AP will be assigned all cases docketed for judge or courtroom 2. Horizontal a. Revolves around each process step rather than individual case b. From one step to another a new prosecutor will take over B. Degree of Control Asserted Over Discretion 1. Assign experienced assistants to major areas of discretionary decision-making and allow basic autonomy 2. Utilize detailed guidelines a. Require assistants to justify decision in writing referencing them b. Require approval by select group of senior assistants C. Discretion Generally 1. Stages involving discretion a. Investigation: whether and how; if GJ used, what evidence should be presented? (i) Is there sufficient evidence to support? b. Immunity: Should some witnesses be granted immunity? (i) Reasons for not subjecting to criminal process? (ii) Conditioned upon participation in diversion program? c. Charging Decision (i) What offense or offenses? (a) d. Motion to Vacate after trial e. Discovery of New Evidence 2. Factors to Consider (ABA 3-3.9) a. Prosecutors reasonable doubt that the accused is in fact guilty b. Extent of the harm caused by the offense (need for retribution deterrence) c. Disproportion of the authorized punishment in relation to the particular offense or the offender d. Possible improper motives of a complainant e. Reluctance of the victim to testify f. Cooperation of the accused in the apprehension or conviction of others g. Availability and likelihood of prosecution by another jurisdiction h. Should NOT give weight to the personal or political advantages or disadvantages which might be involved or to a desire to enhance his or her record of convictions i. In cases which involve serious threat to the community, should not be deterred where juries have tended to acquit j. Should not bring a charge greater in number or degree than can reasonably be supported with evidence at trial or are necessary to reflect the gravity of the offense

3. Reasons to Decline a. Evidentiary problems (i) Search and seizure violation; uncooperative victim; witness unavailability or lack credibility b. Limited resources c. Community (i) Legislative over-criminalization; politically unpopular/lack of community support; jury may nullify; would lead to decreased respect for the law (ii) Discretion guided by effort to discern public needs and community concerns (more aggressive approach on crimes constituents worried about) d. Need for individualized justice; labeling a person a criminal, creating criminal record may do more harm than good (i) May set in motion course of events which will increase probability that person becomes or remains offender e. Other options (civil remedies, diversion programs) available (i) Diversion- program that may not involve incarceration (a) Less serious offense or first felony complaint (b) Counseling, career development, education and supportive treatment services (c) After responding for measurable period court or pros. asked to dismiss prior to trial and adjudication, if fails then resume where left off 4. Selecting the Charge a. Elements b. Evidence c. Victim wishes? (i) Under victim participation model of the criminal process victims should have veto power over charge brought, be able to determine charge, and challenge decision not to charge (ii) Federal (a) Granted right to confer with Govt attorney [18 USCA 3771(a)(5)] Nowhere specified must precede or concern charging decision (iii) State (a) Less than half have const. or stat. provision recognizing right Majority make apparent that consultation about initial charge is not contemplated

D. Limits on Decision Not to Prosecute 1. Judicial Oversight a. Attica: Courts are loath to review prosecutors decisions even if the statutory language makes prosecution mandatory (US Attorneys are authorized and required to institute prosecutions against) because (i) Courts do not want to become superprosecutors (ii) Do not want to compromise secrecy of GJ or accuseds reputation by making evidence file available to adversary (iii) Avoid abuse by persons seeking to have other persons prosecuted (iv) Difficult to establish a standard: when can a prosecutor halt an investigation? What evidentiary standard would be used? How much discretion? b. Cox: Courts cannot force a US Attorney to sign a federal indictment (i) Majority opined that affixing or withholding of signature by US Attorney on an indictment was a matter of exclusive executive discretion that could not be coerced or reviewed by court (ii) Three views (a) GJ finds probable cause but discretionary power of the US Attorney about whether to prosecute may depend on things completely outside of this Since cannot require to affix signature (since member of exec. when making decision not judicial) should not be required to prepare (b) GJ decision to indict should result in US Attorney being required to prepare any indictment they wish to consider or return and sign those that they return May refuse to go forward in open court, not in grand jury room (c) US Attorney must prepare but need not sign indictments Preparation of indictment as drafted with assistance of US Attorney would clearly reflect the conscientious conclusion of GJ When places signature on document, effectively initiates prosecution, thus requiring to do so would block discretion to prosecute as it would require affirmative step to stop it c. Armstrong: Prosecutors have relative competence to assess strength of the case, deterrence value, overall enforcement plan, and enforcement priorities

2. Other possible checks on discretion a. Fed.R.Crim.P. 48(a) (i) Requires leave of court to dismiss existing charges; but this does not give court authority before charge has been brought (Connors) (ii) Protect a against prosecutorial harassment (Rinaldi) b. Criminal Statutes (i) Courts have not interpreted them as mandatory (Attica) c. Grand Jury (i) Most jurisdictions permit GJ to initiate pros. by indictment even though pros. opposes prosecution (a) Some require only that foreman signs (b) Others require prosecutor signature, similar to Rue 7 (ii) In other jurisdictions, GJ cant act on its own because the prosecutor has to sign the indictment (Cox) d. Attorney General (i) In states, AG may bring charge where DA declines to do (Johnson v. Pataki) e. Private Prosecution (i) Rarely available (cf) (a) Individual cannot institute criminal proceeding with pros. approval because SOP violation and DP (People v. Municipal) (b) Private action in the name of citizen for criminal contempt did not violate plea bargain as it wasnt by the state (In re Robertson) f. Special Prosecutor (i) Appointment is invalid as improper interference with prosecutors discretion (Venhaus v. Brown) (ii) Fed. statute requiring appointment of independent counsel to complete investigation and conduct prosecution valid as AG had substantial oversight (Morrison v. Olson) g. Electoral system (i) Public shame and civilian oversight could impact next election h. Internal control in the office may allow supervisor to prosecute where individual DA does not (i) Condition promotion (ii) Pay on charging performance

E. Limits on Decision to Prosecute 1. Arbitrary Prosecution a. May violate equal protection if doesnt pass rational basis test (i) Irrational Distinction (a) No rational basis for enforcing bell on a bicycle law only against prostitutes (Kail) (ii) Desuitude (a) Civil law doctrine rendering a statue abrogated by reason of its long and continued non-use (b) Consistent and growing calls for invocation in academic world, rejected by judges Issues of how long is must be dormant, and statutes charging only rarely occurring crimes (Elliot) (iii) Random Selection 2. Discrimination or Selective Enforcement: Equal Protection Test (Armstrong) a. Requirement for Proving Claim (Must show by Preponderance of the Evidence) (i) D is a member of a protected class (Race: Armstrong/McKlesky, Nationality, Lesser degree gender) -OR(ii) Exercised a fundamental right (1st Amendment: Wayte (vocal draft resistor), Walker (whistle blower), Aguilar (actively opposing govt policy)) -AND(iii) Discriminatory effect (a) Similarly situated persons with this feature were NOT prosecuted -AND(iv) Discriminatory purpose (a) D would not have been prosecuted but for race/gender/exercise of right Statistics not enough: 24/24 cases with black Ds not enough (Armstrong); Statistical trend showing that P seeks death penalty in 70% of cases involving black Ds and white Vs (McKlesky) b. Requirement for Obtaining Discovery (Armstrong/Bass) (i) must make a credible showing of: (a) Discriminatory effect; and (b) Discriminatory purpose c. Selective enforcement, ok: against vocal lawbreakers (Wayte; Ojala) or only grocery stores (Taylor) is an appropriate use of prosecutorial discretion; unless they can show purposeful discrimination d. Improper motives short of discrimination, ok: Annala charging child molester 5 years after the crime ok; Walker enforcing multiple dwelling law only against D who earlier complained about building code violations ok so long as she actually violated a valid law
5

3. Vindictive Prosecution and Due Process a. Where D exercises constitutional right to jury trial pre-trial (Goodwin) no presumption of prosecutorial vindictiveness; D must show actual vindictiveness (Goodwin) (i) Prosecutor may have discovered new evidence or reconsidered; initial 1. Pretrial/no bargaining: decision should not freeze future conduct D must show actual (ii) Proof of actual vindictiveness would establish a violation vindictiveness (iii) Remedy is to get rid of vindictiveness and drop higher charge 2. Bargaining stage: state b. Where D exercise right to go to trial (declining guilty plea), P may increase may not directly retaliate but can forego lesser charges (Bordenkircher v. Hayes) charge for higher one. (i) Guilty pleas essential to justice system 3. Post-trial, presumption (ii) State may deter D from exercising rights but may not retaliate; of vindictiveness vindictiveness permissible if its a choice 4. Remedy is dismissal of (a) Court approved practice of charging low and threatening high if higher charge do not accept plea bargain (iii) Must be probable cause for the higher charges (a) Does not matter if actual vindictiveness since choose to turn down the bargain, and there is standing for charges (iv) Dissent argued that P should have to bring higher charge first and then lower if desired c. Where D exercises right to appeal post-trial, presumption of vindictiveness (i) Prosecutor may not upwardly revise the Ds charge to make the same conduct punishable by a higher penalty (Blackledge v. Perry) (ii) Like Pearce: Judge may not impose harsher sentence to discourage appeals (iii) Presumption dispelled only by explanation that would dispel a reasonable persons belief that charges were motivated by vindictiveness (from standpoint of ) (iv) However, does not preclude if state shows that it was impossible to proceed on the more serious charge at the outset (new evidence coming to light that adjusted interpretation)

II.

Bail and Pre-Trial Detention A. Rationale 1. In favor of bail (Risks) a. Make sure they show up (i) Flight b. Prevent danger to community 2. Against bail a. Presumption of innocence b. Unjust incarceration (i) Imprisoned for longer than sentence c. Need to prepare defense: accuracy, equality d. Imprisonment costs money for the state e. Collateral consequences for detained person and family (i) Job loss, stigma, etc. f. Risks innocents pleading guilty just to get out on time served/probation g. Detention unsafe (i) Illness/ 3. Characteristics a. 28% released on PR; 66% bail set; 6% held without bail b. Timing: of all those released, 48% released within 1 day; 73% within 1 week; 90% within 1 month B. Basic Release Procedure 1. Police decision (low level/misdemeanor cases typically) a. Field release, stationhouse release b. Use of bond schedule 2. Magistrate decision (at first appearance) C. Procedures under Bail Reform Act 1. Personal Recognizance a. Signs agreement to appear in court (i) Subject to condition that no new crimes committed and that person cooperate in collection of DNA sample if authorized b. 2nd most common type of release in US 2. Supervised release with conditions (BRA 3142(c)(B)) a. Agree to special conditions (i) No new crimes (ii) Least restrictive further condition or combination of conditions that may include: (a) Custody of designated person who will assume supervision and report any violation and assure that person will not pose a danger (b) Maintain employment (c) Maintain or commence educational program (d) Restrictions on personal associations, abode, or travel (e) Avoid contact with alleged victim and potential witnesses (f) Reporting (g) Curfew
7

(h) Refrain from possessing firearm or weapons (i) Refrain from excessive alcohol or drug use (j) Property forfeiture agreement (k) Custody following employment, school, etc 3. Unsecured bond a. Conditions and promise to pay if you fail to appear 4. Secured bond set by schedule a. Full cash (i) Pay full amount to court (ii) Returned upon appearance b. Property (i) Sign over lien on house, etc. (ii) Returned upon appearance c. Surety (i) Pay fee up front that is non-refundable to commercial bail agent (a) Criticism Private company determines whether individual will be allowed to go free Bounty hunters granted large degree of leeway in recovering fugitives (non-state actors not subject to const. limitations) Innocent individual paying private companies for their freedom Collusion d. 10% deposit to the court (i) Returned upon appearance 5. Judge may not impose a financial condition that results in detention (BRA 3142(c)(2)) 6. Detention if, after hearing, no combination of conditions is sufficient to assure appearance and safety of community (BRA (3142(e)) a. Rebuttable presumption of detention if: (i) if crime of violence for which a max term of 10 years or more or crime for which life or death is penalty or if accused felon has been convicted of two prior >10 year offenses, or felony with minor victim (ii) Or if Government/court shows that case involves serious flight risk or risk that D will obstruct justice, threaten, injury, or intimidate a prospective witness or juror b. Hearing should afford person opportunity to testify, present witnesses, crossexamine witnesses, present info by proffer (i) Rules of evidence do not apply (hearsay allowed) (ii) Often no counsel c. Consider nature and circumstances of the offense, weight of the evidence, history and characteristics of the person (i) Character, mental and physical condition, ties, employment, financial, length of residence in community, community ties, past conduct,

history relating to drug or alcohol abuse, criminal history and record concerning past appearances (a) D has burden of producing evidence of ties and past record but P has burden of proof concerning likelihood of spearing (Van Atta v. Scott) (ii) Whether person was on probation or parole at time of arrest d. Interlocutory appeal available 7. Jumping bond is a separate crime ( 3146) D. Constitutional Right to Bail- 8th Amendment 1. Incorporated against the states (Schilb v. Kuebel) 2. Bail Clause Anaylsis a. If going to offer bail it must not be excessive (i) No actual right to bail granted by the 8th (Salerno) b. Excessiveness is measured in relationship to purpose (i) Does not limit the permissible reasons for detention (ii) May be detained for other compelling reasons 3. Bail set higher than amount reasonably calculated to assure appearance is excessive under the 8th Amendment (Stack v. Boyle) 4. Fixing of bail must be based on individualized assessment; need for bail cannot be inferred from nature of charge/indictment alone (Stack v. Boyle) a. If unusually high bail is required, Ds rights should be preserved by holding evidentiary hearing 5. Schedule a. Okay if would have had individualized bail determination the next day (Terrell) b. Not okay if hearing to occur three or more days later (Ackies) (i) Concerned that delay would cause individual to accept terms otherwise would find objectionable E. Limits on Preventative Detention 1. Pretrial detention to protect other persons or community at large from criminal conduct by a. Authorized by the BRA b. Fed. pros. have in many cases urged judges not to release people suspected of involvement in terrorist actives even if charged with minor unrelated crimes 2. Substantive Due Process- 5th and 14th Amendment a. Test for when governmental action violates (i) Does action offend some principle of justice so rooted in the traditions and conscience of our people as to be ranked as fundamental? (a) Interferes with rights implicit in concept of ordered liberty (ii) Does it shock the conscience? b. How to determine if regulatory or punitive? (i) Was the legislatures intent to advance a regulatory (non-punitive) goal? (a) Look at similarity to other detention held to be regulatory

(ii) Were restrictions on liberty rationally related to purported regulatory goal, and not excessive? c. Detention without bail does not violate because it is regulatory and not punitive (Salerno) (i) Legislative intent to advance regulatory goal (a) Permissible regulation, not impressible punishment (ii) Fit between statute/action and regulatory goal (a) BRA carefully limits circumstances under which detention may be sought to most serious crimes, grants hearing, maximum length limited by Speedy Trial Act 3. Procedural Due Process (Conduct implemented in fair manner) a. Nothing inherently incalculable about prediction of future violence (Salerno)

10

III.

Screening the Charge A. Function of Screening 1. Primary a. Ensure that there is probable cause for each felony charge 2. Charge Itself a. Provides notice to accused of what he must either contest or admit (DP requirement) b. Protects a against being twice put in jeopardy for the same offense c. No separate notice is required in order to convict a of Y crime when had been charge with X crime, if it is a lesser included offense B. Right to Preliminary Hearing (Information)/Grand Jury Screening (Indictment) 1. Screening obligation (by GJ) under 5th Amendment for Federal government a. Fed.R.Crim.P. 5.1 provides preliminary hearing in all federal felony cases, to be held within a reasonable time following initial appearance, but not later than: (i) The 10th day or (ii) The 20th day following initial appearance if the arrested person is released from custody b. Federal system allows prelim unless, prior to the date fixed for the prelim, an indictment is returned (i) Has essentially eliminated all preliminary hearings because US Attorney can bypass the hearing by taking case to grand jury that sits daily; magistrate could not undermine GJs finding of probable cause 2. Screening obligation not incorporated against States, thus not protected by Const. a. Due process does not require state to adopt the institution and procedure of a grand jury. (Hurtado) (i) Unclear whether dispenses with all forms of independent screening or just that by GJ (challenged procedure required initial determination by magistrate) b. Substantive due process does not protect an individual even against burdens of baseless prosecution without probable cause (Albright) (i) Continuing validity of Lem Woon (direct filing of an information without any examination or commitment by a magistrate did not violate due process) c. States are only required to screen for probable cause if suspect is held in custody following arrest (Gerstein) (i) Preliminary hearing is not required by the 4th Amendment d. All states have some independent screening process: either GJ, prelim hearing bindover, or direct filing (great variation outside of basic judicial review) (i) 18 states require indictment for all felonies; prelim hearing within a specified period after arrest (but can be bypassed by indictment) (ii) 32 states permit felony prosecution to be brought by information (a) Prosecutors overwhelmingly choose to proceed by information (see section below) unless prelim would have protracted evidence, would lead to key defense discovery, or would force victim to testify too many times
11

(b) A handful of information states allow direct filing: proceed without prelim 3. Reasons for State Provided Screening (even though not Const. required) a. Saves resources/cheaper (i) Short and thus weed out cases with no basis prior to long trial b. Preview Witness Credibility (i) Allows better prep of witnesses (pro-P); put witness to the test of testifying c. Promoting Vs interest by presenting issue in a public forum d. Gaining D perspective as to events e. Witness ID of the subject f. Promoting public confidence in a sensitive prosecutorial decision g. May help P get to plea faster (pro P) h. Preserves testimony and Ds cross-examination protections for witnesses later unavailable for trial (pro-P) (i) Does not violate 6th Amendment Confrontation Clause (Green) if had adequate opportunity to cross-examine (Crawford) (a) Some state courts have imposed further limits as a matter of state law 4. Right to counsel in prelim, not in GJ (Coleman) a. Prelim is critical stage (i) Cross exam of critical witnesses (ii) Cement testimony for trial (iii) Discover statess case (iv) Early opp for psych exam or bail argument 5. Minimum constitutional requirements:
Preliminary Hearing or other Screening 1. States may use either Prelim or GJ to screen (must be impartial adjudicator) Grand Jury 1. Federal courts are required by 5th Amendment to screen charges through grand jury; states are not required 2. Where GJ charges - If person is not in custody, no screening under 4th Amendment - Even if in custody, GJ counts as screening, no need for additional screening 3. No right to counsel in the GJ

2. Where prosecutor charges without GJ:

- If D is in custody before trial, 4th Amendment requires magistrate to do ex parte screening (Gerstein) - If D is not in custody, no right to prelim or magistrate screening
3. Where prelim given, right to counsel (Coleman)

12

C. Preliminary Hearings 1. Purpose: a. Determine whether there is sufficient evidence to bind the case over (i) Only between 2% and 33% of cases dismissed on prelim: but this number could depend on level of prosecutorial screening, utilization of GJ, caseload at trial level, use of plea bargains, whether D regularly insists on prelims, and whether P presents all key witnesses or just enough for standard b. D typically seeks other advantages of prelim unless the states case is actually weak (but may be advantageous to force a bench trial where states case is weak) 2. Rules a. Bindover Standard (i) Reasonable belief that offense committed and that D committed (Clark) (a) Must present sufficient evidence Relatively low threshold due to assumption that case will only get stronger as investigation continues (b) Magistrate must view all evidence in light most favorable to pros. and draw all reasonable inferences in favor of the pros. Magistrate may not weigh credibility, sifting and weighing the evidence is left to the fact finder at trial (Hunter; Colorado) [some states allow] (c) NOT a rubber stamp though b. Evidentiary Rules (i) Recognize testimonial privileges (ii) Brady duty to turn over exculpatory evidence also applies (Mitchell) (iii) States vary on applicable rules of evidence (a) Some require full application Will not reject a bind over, however, because magistrate relied erroneously on incompetent evidence (b) Some more partially apply, allowing certain types of inadmissible evidence to be considered (c) Majority state rules of evidence do not apply (iv) Hearsay typically permitted; some states only allow limited hearsay (a) Many witnesses will give evidence on matters not really in dispute thus undue burden for them to testify at both (b) Less likely to assist police if believe that be required to make appearance at both proceedings (c) GJ allowed to consider in most jurisdictions, thus stricter stnd. will encourage pros. to bypass preliminary hearing by obtaining indictment (d) Magistrates are sufficiently familiar with limitations to appropriately judges its degree of reliability
13

(v) Many states do not recognize exclusionary rule for unconstitutionally obtained evidence (even those which hold all evidentiary rules applicable) c. D has right to counsel because it is a critical stage (Coleman v. AL) d. If charge is dismissed, states are split on granting pros. an appeal of right to contest; where appeal not provided likely to try to: (i) Seek GJ indictment (a) May indicate despite magistrate refusal (need not even be informed usually) (ii) Refile charges and seek another hearing on the same evidence (a) Likely subject to powerful institutional restraints Smaller districts might be unable to have hearing before another magistrate Even if in front of another magistrate may be hesitant to reach result inconsistent with first ruling, calling attention to differing perspectives of members of same bench (b) Minority of states prohibit Typically provide for pros. appeal and consider it to be only appropriate avenue Do allow refilling if pros. has new evidence (Jones v. State); differing stnds. 3. Challenges a. Types of challenges (i) Denial of right to prelim (by accepting a faulty waiver) (ii) Magistrate erred in finding probable cause (iii) Erred in procedural ruling at the prelim b. Approaches to Assessing Error (i) Coleman approach (federal and some states): vacate conviction and remand for lower court to determine whether denial/error was harmless under Chapman; is D able to point to specific aspect of the trial where D was adversely impacted by having lacked counsel at prelim (a) State courts tend to assume error was harmless and require D to prove harm (ii) Per se harmless approach: most state courts (a) Like the GJ standard in Mechanik (iii) Jurisdictional defect approach: cannot be cured by subsequent trial and conviction = automatic reversal (a) Absence of proper bindover comparable to not filing a charge

14

D. Grand Jury Basics 1. Procedure (FRCrP 6) a. Indict on probable cause b. No judge or magistrate presiding c. GJ cant go forward without the US Attorney d. GJ transcripts are not public records e. GJ can ask a question of a witness as well as the prosecutor f. D/target does not have a right to counsel at GJ or to review charge (i) Even though preliminary hearing is a critical stage GJ review is not g. Double jeopardy does not attach; if GJ refuses to indict, prosecutor may try again h. Jurors, interpreters, court reporters, transcriber, or attorneys are sworn to secrecy; witnesses do not have secrecy obligation (FRCrP 6(e)(2)) i. D can waive right to GJ and proceed by information (particularly if D is in jail and would be prejudiced by delay) 2. 5th Amendment ensures that federal charge for a felony offense will not be brought without protection of review and acceptance of charge by grand jury 3. Secrecy Rationale (US v. Procter & Gamble) a. Prevent the escape of those whose indictment may be contemplated (i) Do not want to tip someone off that they are being investigated b. Prevent persons subject to indictment or their friend from importuning the grand jurors (i) Ensure freedom in deliberation c. Prevent subordination of perjury or tampering with the witnesses who may testify before GJ and later appear at trial (i) Encourage free disclosure by witnesses (given under oath but if no indictment or trial it remains unknown) d. Protect the innocent accused who is exonerated from: (i) Disclosure of fact that had been under investigation can be damaging (ii) Expense of standing trial where no probability of guilt 4. Cannot seek civil remedies against GJ witness as it would compromise the secrecy of the proceedings 5. Effectiveness of GJ a. Three Groups (i) GJ is worthless rubber stamp of the prosecutor (a) Either eliminate it or give option to choose prelim. hearing over GJ (ii) GJ has potential as legitimate screening alternative to prelim. hearing if safeguards put in place (iii) GJ is highly effective screening agency b. Suggested reforms (i) Requiring court to fully inform jurors of independent authority (including nullification) (ii) Giving GJ its own counsel or ask court for legal advice (iii) Giving target the right to testify

15

(iv) Requiring prosecutor to present available exculpatory evidence Forbidding use of unconstitutional evidence (v) Prohibiting use of hearsay (vi) Requiring prima facie cases standard of proof (vii) Requiring affirmative vote of a supermajority of grand jurors (viii) Prohibiting resubmissions following decision not to indict (ix) Challenge procedure that requires court to review transcript c. Assessments of reforms (i) Group 1 (a) Basic problem is in very structure of grand jury and reform not enough (b) Lay persons applying unfamiliar standard to one-sided presentation of the facts (ii) Group 3 (a) Valuable sounding board for prosecutors (b) Brings laypersons sense of reality; strength is in controversial cases E. Evidentiary Limits 1. D may not challenge indictment based on hearsay evidence, insufficiency of the evidence, or incompetency of the evidence, if an indictment, on its face, is valid that is enough (Costello) a. Allowing these challenges would create impermissible delay and create minitrials b. History of allowing broad range of evidence into GJ (see Holt v. US) c. Allowing challenge would harm benefits of secrecy (i) But secrecy concerns arent as potent because can choose to abrogate secrecy that protects his reputation, witness has already been honest, jurors cant be influenced 2. May not challenge based on unconstitutionally obtained evidence (Calandra) 3. USAO manual directs attorneys: a. Not to present evidence he knows was obtained as a direct result of the constitutional violation b. To utilize hearsay but present it to the GJ as such 4. Remedies a. Hearsay: No relief (Costello) b. No evidence: no relief (Costello) c. Failure to present exculpatory: No relief unless there is a long-standing patter or common problem in the district (Williams) d. Illegally obtained evidence: No relief (Calandra) F. Grand Jury Selection 1. Vicinage (geographic area) a. Boundary from within jurors are selected 2. Jury List from which Selected a. Voters/tax/drivers 3. Qualified Wheel/Pool

16

a. After questionnaire weeds out those from list who are not qualified (English, non-felons, age) 4. Venire (array) a. Those summoned for certain jury (less those excused/deferred, high number due to length of time GJ sits) 5. No regularized procedure for participation of defense counsel and prosecution in selection process G. Challenging the Grand Jury 1. Composition a. Standing 1. Basis for (i) Third-party standing where D has stake in the outcome, jurors have challenge little opportunity to raise the challenge, and D shares interest in the outcome a. Discrimination in (a) Even white D has standing to challenge exclusion of black petit composition: and grand jurors (Powers + Campbell) automatic reversal b. Discrimination in selection of GJ (Rose) (i) Forbidden by the Equal Protection Clause and the Statute (a) Under Equal Protection must show purposeful/deliberate b. Constitutional discrimination in selection of the venire (array) (structural, Make a prima facie showing which shifts burden to discrimination, pervasive state to show discriminatory impact was not purposeful problem): presume (Rose; Vasquez; see relief below) prejudice (Isgro) (ii) Batson challenges for composition of the grand jury allowed, but far less common in GJ c. Rules/Statute: c. Discrimination in selection of foreperson Must raise before (i) Forbidden by the Equal Protection Clause trial and show d. Personal bias of grand juror does not require reversal (Rule 6(b) doesnt give significant influence on outcome (B of power) Nova Scotia) (i) Bias theory of the functions of a grand jury, does nor require that 2. Secrecy grand jurors should be impartial and unbiased (Knowles) Concerns (ii) Substantial minority of state provide statutory bias objections: relatives of the victim, state of mind prevent impartiality, (a) must make preliminary showing of likely prejudice to invade secrecy of GJ transcript (iii) In most jurisdiction pre-indictment publicity has been held to not be a violation (a) Prejudicial pre-indictment publicity cannot justify the dismissal of an indictment since lack of impartiality of a GJ is not ground for dismissal. (Jones v. State) Would create many administrative difficulties (b) Argument for dismissal on basis of prejudicial pretrial publicity in appropriate case where showing of actual bias on part of seated juror is widely seen as inevitably doomed as a matter of law has no right to voir dire and GJ may not testify as to deliberations, thus no way to prove
17

e. Remedy (i) Discrimination on the basis of race during selection of GJ results in automatic reversal of the conviction, even if D was later convicted by fairly-selected jury (Rose) (a) Vasquez: Even on habeas, relief can be granted because GJ has power to charge greater/lesser offense, numerous or singular counts, capital or noncapital offense (b) Nothing more damaging than discrimination (c) Too hard to show if discrimination made a difference 2. Prosecutorial Misconduct (Three Types) a. Constitutional Violation (i) Where structural protections have been so compromised as to render proceedings fundamentally unfair, allow presumption of prejudice to the D (Isgro) (ii) But no constitutional error where charging judge told grand jurors (1) not to judge the wisdom of the law and (2) that P is presumably acting in good faith (Navarro-Vargas) (a) Does not undermine jurors independence b. Statutory or Rules violation (FRCrP 6) (i) Dismissal only if error had substantial influence on outcome of the proceeding and it is challenged prior to conviction (a) Having two witnesses in at the same time (Mechanik) (b) Evidentiary violations rarely rise to the level of misconduct unless it is a pervasive problem (Williams) c. Ethics, Norms, Internal Guidelines violation (i) US Attorneys Manual prohibits making argument about Ds silence d. May not allege prosecutorial misconduct for failure to present exculpatory evidence (i) Courts have no supervisory power over the GJ and cannot dismiss indictment for failure to present exculpatory evidence to the GJ (Williams) (ii) Rationale (a) GJ is functionally independent and not bound by constitutional rights like double jeopardy, right to counsel or procedural rights (exclusionary rule, hearsay rules) (b) GJ is accusatory not adjudicatory (c) Could lead to D presenting a defense (iii) Some evidence claims may still exist after Williams (a) If D demonstrates that misconduct is long-standing or common problem in the district and Ps actions caused actual prejudice for the D (Boettcher) 3. Remedies for Prosecutorial Violation of Statute or Rule a. To gain court ordered discovery to prove misconduct in the GJ (i) D must show that a ground may exist to dismiss the indictment because of a matter that occurred before the GJ (Rule 6(e)3(E)(ii))

18

(a) Must produce affidavit of friendly witness or portion of transcript otherwise released as there is no const. right to exculpatory evidence (Bank of Nova Scotia) (b) Very difficult to do as must rely on preexisting rules of grand jury, which tend to uphold secrecy, sharply restricting access H. When Error Requires Relief If Raised PRIOR To Trial 1. Harmless Error Rule applies (Fed. Rule 52(a)) [Govt] a. Statutory Violation (i) Dismissal unless govt show error has no substantial influence on outcome of proceeding b. Const. Violations (i) Dismissal unless govt shows beyond a reasonable doubt that error did not affect outcome of proceeding 2. Thus Must show error was not harmless/prejudicial impact (Bank of Nova Scotia) a. Dismissal appropriate only if it is established that the violation substantially influenced the GJs decision to indict or if there is grave doubt that the decision to indict was free from substantial influence of such violations 3. Judge will probably just take motion under advisement until after conviction

19

IV.

Joinder A. Overview 1. Multiple charges against the same in same indictment/trial a. Statutory Limits (i) Rule 8 (ii) Rule 14 (Pereira) b. Const. Limits (i) Double Jeopardy (Dixon; Ashe) 2. Multiple s in the same indictment/trial a. Statutory Limits (i) Rule 8 (Schaffer) (ii) Rule 14 (Zafiro) b. Const. Limits (i) Bruton/Gray B. Joining Offenses: Which May be Joined 1. Joining: Rule 8(a): the indictment or information may charge a D in separate counts if offenses are a. Of the same or similar character (i) Based on offenses charges, location and time, whether occurred within close time span, similarity of victims, M.O. of the offenses (a) Allowing offenses that took place at a different time and place is controversial b. Are based on the same act or transaction (i) Look for time between offenses usually limited to days or even hours c. Are connected with or constitute parts of the common scheme or plan (i) Not a question of overlapping evidence; look for factors connecting two offenses: embezzle to cover up fraud, steal a car to use after a robbery d. Reasons to join (i) Economy (ii) Finality for the D (iii) Constitution may require (iv) Easier on the witnesses (v) Concurrent sentences 2. Severance: Rule 14: If joinder of offense appears to prejudice a D or the government, the court may order severance of counts (not a matter of right) a. Question of whether the trier of fact will be able to distinguish the evidence and apply the law intelligently (i) If motion denied, reviewed under abuse of discretion standard (a) Common support for finding that was not prejudiced Jury is capable of following the judges instructions to ignore certain evidence or to consider certain evidence only as to some of the charges

20

1. Can charges be joined under Rule 8? 2. Should they be severed under Rule 14? a. Spillover prejudice i. But would evidence come in anyway? ii. Are crimes simple and distinct? b. Inconsistent defenses: evidence on one charge is weak and he wants to testify on the other; is it distinct from other claims for severance?

If the jury has acquitted the on any count this shows that the jury has been selective and thus must have kept the evidence separate The had no ground for complaint if he was convicted of the several counts but received concurrent sentencing That any prejudice form the joined is cured by overwhelming evidence of guilt b. Spillover prejudice (Drew): If charged with multiple crimes, jury may infer criminal disposition; worse crime makes D appear worse than if only minor crime charged (i) But no severance required if: (a) Rules of evidence would allow the jury in the trial of charge A to hear evidence of charge B anyway, even if two were tried separately [404(b)] Motive, intent, absence of mistake, common scheme or plan where proof of one tends to establish the other, identity of the person charged with the commission of the crime (b) Even if evidence of charge B would not be admissible, if the two offenses are simple and distinct Likelihood of jury confusion; does P refer to crimes at the same time or use them interchangeably c. Inconsistent/Conflicting defenses (Cross): Forces D to choose between inconsistent defenses (i) D must proffer specific testimony that he wants to give in one case and specific reason why he doesnt want to testify in the other; otherwise any D charged with multiple counts could move to sever (ii) Court should balance: evidence against D for each offense, availability of defense evidence other than proffered testimony, plausibility of testimony, effects of demeanor impeachment and cross (iii) Notoriously difficult to apply before trial; appellate review standard is abuse of discretion d. Jury cumulates evidence of the various rimes charged and finds guilt when, if considered separately, it would not find so

3. Const. Limits (What Must Be Joined) a. Fifth Amendment Double Jeopardy (i) No person shall subject for the same offence to be twice put in jeopardy of life or limb (a) Applies to both successive punishments and to successive prosecutions for the same criminal offense b. Claim Preclusion (i) Same offenses must be joined or DJ will bar prosecution for the 2nd offense
21

1. Same series/transaction, character, or common scheme/plan? Rule 8 2. May the court sever: spillover prejudice or inconsistent defenses? 3. Must offenses be joined? Same elements? If different V, fact necessarily determined? 4. Std on appeal for misjoinder

(ii) Double Jeopardy (a) Does each offense have one separate element not contained in the other? (Dixon-Blockburger) If not, then the leg. presumably intended them to be the same offense (alternatives of one another) and thus that person can only be punished with one or the other, not both If one offense is a lesser-included, barred by double jeopardy from bringing separate prosecutions Presumption can be overcome by very clear evidence that the leg. intended the two offenses to be punished cumulatively (see Missouri v. Hunter) Dixon: Contempt + drug crime = same offense; contempt (for drug crime) and assault with intent to kill = not the same offense, intent to kill is an extra element

(b) Waiver If D seeks to sever under Rule 14 because he wants to avoid prejudice, he is deemed to have waived double jeopardy c. Collateral Estoppel/Issue Preclusion (i) Distinguish from DJ: different victim means its not the same offense; look to claim preclusion (ii) Acquittal for one offense will bar a later prosecution for a different offense if a FACT necessary for conviction of the second offense was necessarily determined earlier in the s favor BY a final and valid judgment, after parties had full and fair opportunity to litigate in prior case (Ashe) (a) Court must examine the record to see whether a rational jury could have grounded its verdict upon an issue other than that which the seeks to foreclose from consideration (Ashe) (b) Was the fact fully-litigated? (c) Look at charges to the jury, arguments to the jury, questions by jurors (d) If D conceded all other issues, acquittal was probably based on that issue C. Joining Defendants 1. Rationale a. Pro: Economy b. Con: Prejudice, constitution may require severance; joinder may compromise confrontation rights 2. Statutory Limits a. Rule 8(b): Two or more defendants may be charged together if: (i) Same act or transaction (ii) Same series of acts or transactions
22

(a) But even if the conspiracy charge is dropped, can still be charged together, judge must consider prejudice (Schaffer) (iii) Note: no similar character provision (a) Even if closely-related in time or have overlapping evidence, different offenses cant be joined unless they are part of the same act or transaction b. Rule 14: if joinder appears to prejudice D, should be severed (typically argued by least culpable D) (i) Complexity of the evidence is such that the trier of fact probably will be unable to distinguish the evidence or apply the law intelligently (a) Question is whether it is within the jurys capacity, given the complexity of the case, to follow admonitory instructions and keep separate, collate and appraise the evidence relevant only to each defendant (Warner) (b) Where proof is carefully compartmentalized and there is no prejudice, joinder not an error (Shaffer) (ii) Because several defendants have antagonistic defenses (a) Not entitled to severance merely because they have a better chance of acquittal separately or have conflicting defenses; only if serious risk that join trial would compromise a specific trial right or prevent the jury from making a reliable judgment (Zafiro) Evidence that jury should not consider against one that would not be admissible if D tried alone But just because one D is capital and the other is not, not entitled to severance (iii) Otherwise impossible to call a co-D as a witness (a) Must show that he would call the co-D at a severed trial, that the co-D would testify, and that the testimony would be favorable to the D (Substance of testimony and exculpatory effect) (Nguyen) c. However, failure to sever (even clear error) can be harmless (Lane) 3. Constitutional Limits a. Confrontation: Cannot introduce co-Ds confession at joint trial when co-D doesnt testify and the statement directly implicated the (Bruton) (i) Limiting instruction not sufficient to reduce prejudice (ii) Removing name from co-Ds confession is not sufficient to remove prejudice (Gray, jury could immediately make inference as to who deleted name implicated) (iii) Must sever trial, provide separate juries, or eliminate any reference to anyone other than co- from confession (Richardson: confession didnt suggest Ds presence, inferred from later evidence) D. Analyzing 1. Multiple defendants first a. Identify the least culpable defendant (the person alleged to have committed the least serious crimes); then
23

b. See if he has a basis for claiming misjoinder with the others under Rule 8b; then c. Look to see if there is prejudice from joinder with the others under Rule 14 d. Constitutional basis for severance (e.g. Gray?) 2. Multiple offenses second: (once youve decided who must be tried separately, examine the offenses against each separate defendant): a. Basis for misjoinder of offenses under Rule 8a? b. Basis for severing properly joined offenses under Rule 14? c. Constitutional basis to join/bar? DJ or CE?

24

V.

Regulating Delay A. Interests 1. Advanced by speedy trial a. s interests (i) Prevent long detention (a) Even if believe will be convicted (will start servicing sentence; be moved from jail to prison) (ii) Preserve witness and evidence (iii) Certainty/Finality (a) Psychological, social pariah (iv) Minimize costs if found guilty (repayment), or if not (loss of jobs) b. Society (i) Costs to taxpayers, families, employers from incarceration (ii) Released could flee/harm others (iii) Exonerates innocents faster (iv) More accurate verdict (a) Evidence is fresh and witnesses available (v) Reduces undue bargaining leverage (a) through overcrowding (b) P through use of inability of to make bail (vi) Delay undercuts rehab (a) In very violent, unstable environment (jail) (vii) Victims interests (viii) Reduces risk of vigilante justice 2. Advanced by delay a. Ds Interests (i) More time to build case (ii) Negotiate plea instruments (a) Large backlog of cases leads to easier ability to plea to lesser offense (iii) Case may fall apart or weaken (a) State has burden of proof and evidence lost may be crucial to the case (iv) Put off eventuality of jail (a) Out on bail (v) Prejudice might die down (a) More likely to find impartial jury the longer separated from news of the vent b. Society/Prosecution (i) Time to develop proof (ii) Increased leverage for settlement (iii) Other more important cases

25

B. Constitutional Limits 1. Delay between Crime and Charge (Prior to charge) a. Primary protection is the Statute of Limitations on a crime (i) Prevents stale evidence/inaccuracy (ii) Risk of recidivism diminishes with time (a) If has committed new crime can be charged with those offenses, if not need for protection is less compelling (iii) Community desire for retribution gives way to compassion with time (iv) Cabins blackmail risk (a) Threat to prosecute or disclose evidence to law enforcement (v) Promotes repose (a) Through security and stability of human affairs b. Due Process (Lovasco) (i) Not 6th Amendment regulation (ii) Dismissal requires showing of (a) Prejudice Defense impairment Risk of influence on outcome of proceeding (b) Delay was deliberate effort by pros. to impair defense, or reckless disregard of a substantial risk that delay would impair (Marion- Prejudice is a necessary but not sufficient element of DP claim must consider reason) Hoping for impairment of defense is not an acceptable reason Some lower courts require showing of bad faith c. Forcing P to bring charge before it is ready risks (reason why second prong of test) (i) Impairing ability to fully investigate (ii) Impairing ability to consider wisdom of charges (iii) Resolving doubtful cases in favor of prosecution 2. Delay between charge and trial: a. Sixth Amendment Speedy Trial Clause (i) In all criminal prosecutions, the accused shall enjoy the right to a speedy and public trial (ii) Violate if delay is: (a) Long enough to trigger constitutional scrutiny (triggering mechanism) Lower courts: 6-10 months (b) Fails the Baker Balancing Test

26

b. Test: Barker balancing (i) Length of delay (a) Minimum time necessary to trigger is at least 6-10 months (b) Time beyond that weighed in the balance (ii) Reason for delay (a) Good reasons include witness availability, overcrowded dockets (iii) Whether asserted the right (demand) (a) Failure to assert weighs against the (iv) Prejudice to (3 types) (a) Impaired defense, incarceration, and anxiety/reputation (v) Delay caused by or counsel weighs against the (regardless of if counsel is private or publicly assigned) (a) Public defenders failure to move the case forward is not attributable to the state (Brillon) Not absolute, as delay caused by systematic failures of public defender system could be attributed to State c. Applying the test (i) Barker (weighs in favor of pros.) (a) never asserted and didnt show prejudice But had he shown these, the Ps reasons for waiting were unacceptable (ii) Doggett (weighs in favor of ) (a) 8 year delay clearly suffices (b) Reason for delay was government negligence No serious effort to locate , if there had been he was easily locatable (c) D cant be penalized for failing to assert right prior to arrest because he had no knowledge of the indictment (d) No showing of impairment Presumption of delay causing impairment was advanced nor rebutted, thus no weight d. Remedy for Violation (i) Dismissal with prejudice only possible (a) Remanding would only add to delay (Strunk) C. Statutory Regulation: Speedy Trial Act 1. Supplements constitutional regulations a. Passed in 1972 over objection of both DOJ and Judicial Conference b. Effective as of 1980 2. Most states have statutory regulation that is roughly similar 3. Basic Analysis a. Beginning date: When does the clack start? Did it start over at any point? b. Isolate periods when clock is running (including continuances) c. Determine any periods when clock stopped [excludable under (h)] d. Total up days when clock running e. If violation, what is the remedy?
27

4. Starting the Clock a. From arrest or summons, 30 days until indictment or information (i) Unless no GJ has been in session during such 30 period (3161(b)) b. From public filing of indictment or information or date of first appearance before an officer of the court, whichever occurs later, trial must occur within 70 days (3161(c)(1)) (i) Unless D consents in writing, trial shall not commence less than 30 days from first appearance (3161(c)(2)) 5. Restarting the clock a. seeks to dismiss, granted, and then new charge (i) If indictment or information is dismissed on Ds motion or charge dismissed and new complaint is filed charging D with the same offense or offense based on the same conduct or arising from the same criminal episode, starts the clock over and trial must begin with 70 days (3161(d)(1)) b. Successful interlocutory appeal (i) If indictment or information is dismissed by trial court and reinstated following appeal, trial must occur 70 days from day decision becomes final or 180 days if witnesses are unavailable or other factors from passage of time (3161(e)) c. Mistrial or grant for new trial (i) If retried after mistrial, 70 days from date of action occasioning the retrial (3161(e)) d. Deemed indicted from day earlier plea is withdrawn (3161(i)) 6. Isolate periods of excludable delay (carve up into separate causes) a. Periods attributable to (3161(h)(1)) (i) Proceeding including examinations to determine mental competency of physical capacity of (ii) Trial with respect to other charges against (iii) Interlocutory appeal (iv) Pretrial motion, filing through conclusion or other prompt disposition of the motion (a) Lower courts interpret as 45 days (v) Proceeding relating to transfer or removal to another district under FRCP (vi) Transportation of from district to exam or hospitalization, except time in excess of 10 days from date of order of removal or transport and s arrival (presumed unreasonable) (vii) Consideration of the court of proposed plea agreement (viii) Period (less than 30 days) any proceeding by is actually under advisement by the court (ix) Deferred by government by agreement with (x) Absence or unavailability of the or an essential witness (a) If whereabouts unknown and he is attempting to avoid apprehension or prosecution (absent); or whereabouts known

28

but cannot be obtained by due diligence or he resists appearing for trial (unavailable) (xi) Period where is mentally or physically unable to stand trial b. Information or indictment is dismissed on motion for the P and thereafter a charge is filed against the for the same offense delay would run for new charge as though there had there been no previous charge (i) Compare to motion by : clock restarts; here, clock stops and then continues to run at same spot c. Where is joined for trial with co- for whom time has not run and no motion for severance has been granted (i) Co-s excludables also excludable d. Continuance granted by judge sua sponte or at request of the if judge granted such continuance of the basis of finding that the ends of justice outweigh interest in speedy trial; court must set forth reasons for finding. (i) Consider: (a) Would failure to grant make continuation of proceeding impossible or result in miscarriage of justice (b) Whether case is unusual or complex due to number of D, nature of the prosecution, novel questions of fact or law, otherwise unreasonable to expect adequate preparation (c) Unreasonable to expect indictment in 30 days from arrest (d) Would deny D reasonable time to obtain counsel, deny D or government continuity of counsel, deny necessary time for effective prep (e) No continuance for lack of diligent prep or failure to obtain available witnesses by government (ii) may not prospectively waive application of the Act so as to make a continuance effective notwithstanding the failure of the judge to place the necessary findings on the record (Zedner) e. Any period of delay (but less than 1 year) ordered by court by preponderance of evidence for evidence in foreign country f. See 3161(j) regarding special rules for D in custody 7. Calculate time: if over time period, violation 8. Remedy a. Dismissal (i) Arrest-indictment more than 30 days (ii) Charges filed/first appearance trial (a) D has burden of proof supporting the motion but P has burden for evidence relating to unavailable witnesses (iii) D is detained under Bail Reform Act: no detainee shall be held in custody pending trial after 90 day period b. Dismissal without prejudice depending on: (i) Seriousness of offense (ii) Facts and circumstances leading to dismissal (iii) Impact of re-prosecution on STA and administration of justice (iv) D should argue prejudice if dismissal is P or judges fault
29

c. Sanctions for D or P counsel (i) If: (a) Knowingly allow cases to be set without disclosing that witness will be unavailable (b) Filing frivolous motion for purpose of delay (c) Knowingly false and material statement for the purpose of continuance (d) Willfully fails to proceed to trial without justification (ii) Type of sanction (a) Fines: Ds appointed counsel, reduced compensation; Retained counsel, of compensation; P, fine up to $250 (b) Deny right to practice up to 90 days (c) Report with disciplinary committee

30

VI.

Right to Counsel A. Sources of Right 1. Federal Const. a. 5th Amendment- Miranda/Edwards (during custodial interrogation to protect privilege against self-incrimination) b. 6th Amendment- Assistance for trial c. Equal Protection- Counsel on Appeal for Indigent d. Due Process- Counsel on Appeal 2. State Const. 3. Statute a. Capital cases, post conviction 4. Rule a. FRCrP 44- Indigent is entitled to have counsel appointed to represent at every stage of proceeding from initial appearance through appeal, unless waives this right B. Incorporation against States by 14th allowed Courts to recognize plea bargaining and settlements as legitimate and not coerced C. Road to Gideon 1. Powell v. Alabama a. Initial decision to find DP right to counsel in capital cases 2. Johnson v. Zerbst a. 6th Amendment right to appointed counsel in federal felony cases if indigent 3. Betts v. Brady a. DP requires state to provide counsel when there are special circumstances (illiteracy) b. Approach survives post-Gideon in determination of right to appointed counsel in proceedings other than criminal prosecution (probation/parole revocation, etc.) D. Constitutional Right 1. When Does Right to Counsel Apply? a. Generally (i) During critical and adversarial stages of trial (a) Trial like confrontations where lawyer acts as a spokesman for, or advisor to the accused (b) What is? Pretrial corporeal identification (Wade); Police questioning (Brewer); Certain kinds of arraignments and preliminary hearing (Coleman); Pretrial motions (e.g. to suppress); Plea; Trial; Sentencing (c) What isnt? Probable cause hearing (Gerstein); Pretrial photographic identification procedures (Ash); GJ investigation (d) Bail hearings is debated among states

31

b. Pre-trial (i) No right to counsel prior to adversarial proceedings (Kirby) (a) No right to counsel in the GJ (b) No right to counsel in pre-indictment line-p or photo identification line-up (c) During interrogation, 5th Amendment guarantees right to counsel in order to protect 5th Amendment rights (Miranda) th c. 6 Amendment Right to counsel attaches at first judicial proceeding (i) Formal charge or when government has used the judicial machinery (formal hearing, prelim hearing, indictment, information or arraignment) to signal a commitment to prosecute (Rothgery) (a) Where accusation is filed with a judicial officer and prompts restrictions on the accuseds liberty in order to facilitate prosecution (ii) Does not mean entitled right away, merely that must be appointed within a reasonable time after attachment (a) Only entitled to assistance at critical stages d. Right to Counsel at Trial (Gideon v. Wainwright) (i) 6th Amendment guarantees right to the aid of counsel at trial (a) Bright line rule is less subject to abuse by states than special circumstances (ii) The right is fundamental and is incorporated against the states through the 14th Amendment (iii) Advances key concerns of criminal justice (a) Reliability- Protecting against conviction of innocent (b) Equality- Protect poor and minority s (iv) Required for enforcement of other rights e. Right to Counsel During Appeals (i) Basis (a) Under equal protection, it would be discrimination for wealthy s to have appeals heard while indigents do not (Douglas) 6th Amendment does not apply to appeals (ii) Discretionary Appeal (a) Right to counsel if first-tier appeal (Halbert) If forced to act pro se will face record unreviewed by appellate counsel, and have not attorneys briefs prepared for court review (b) No right if to State Supreme Court or US Supreme Court (Ross) 14th Amendment does not require absolute equality and it does not equalize economic conditions Douglas right will have been met as will have had aid of counsel during first appeal, a transcript of trial, appellate brief on his behalf, and an opinion from the first court of appeals Appeal is a sword to upset prior determination of guilt, not a shield to protect against being haled into court
32

(iii) No right to counsel post-conviction at collateral review (Giarratano/Finely) (a) i.e. Petition for state court relief, then petition for federal habeas 2. For What Types of Cases Is Counsel Required? a. Felony cases (i) Counsel always required (Gideon) b. Misdemeanor cases (i) Only if receives sentence of imprisonment (a) No person may be imprisoned without right to counsel (Argersinger) (b) Right to counsel only if actually imprisoned (Scott) If crime only carries a risk of incarceration, not required to appoint counsel (if no counsel, cant sentence to jail) (c) Suspended sentence requires counsel, as it is a prison term imposed for the offense of conviction (Shelton) If violates terms of probation, he will be imprisoned for the underlying offense, not the violation Counsel at probation revocation doesnt help because the question is only whether he violated, cannot challenge underlying, original judgment (d) Even if sentence is time served, right to counsel Incarceration is part of the sentence (ii) Judge must determine early on (when right to counsel attaches) whether to assign counsel (a) If does not then option of imposing a sentence of incarceration is foreclosed
Investigation 1st. Appearance or Indictment 6th Amendment right to appointed counsel attached Rothgery Preliminary Trial or Plea Appeal PostConviction Review

No 6th Amendment right to appointed counsel -May be 5th Amendment right Miranda Edwards

6th Amendment right to appointed counsel Coleman

6th Amendment right to appointed counsel Gideon Argersinger/ Scott

Equal No right to Protection Right counsel & Due Process Finley to appointed counsel Douglas Halbert Ross (no right for discretionary review)

33

E. Waiver and Self-Representation 1. There is a historical and constitutional right to represent oneself at trial (Farretta) a. Protects autonomy and right to craft his own defense b. Waiver must be knowing and voluntary (i) For Trial (a) Totality of the circumstances test Made aware of dangers and disadvantages (b) Record must show that an accused was offered counsel but intelligently and understandingly rejected the offer (Carnley v. Cochrane) (ii) For Plea (Tovar) (a) need only be informed of: The nature of the charges The right to be counseled regarding the plea; and, The range of allowable punishments (b) Need not inform that he risks overlooking viable defense (c) Forgoes independent opinion about wisdom of plea 2. Limits a. No constitutional obligation to inform D of right to proceed pro se in absence of clear indication that he wishes to proceed without counsel b. D must: (i) Clearly and unequivocally waive right to counsel (ii) Timely assert right c. Competency for going to trial with lawyer (Dusky/Drope) OR for pleading guilty without lawyer and waiving counsel for plea (Godinez) (i) Rational as well as factual understanding of the proceedings (ii) Sufficient present ability to consult with his lawyer with a reasonable degree of rational understanding d. Competency for going to trial without lawyer (Edwards) (i) State may limit a Ds right to self-representation by insisting on representation when D is not mentally competent to conduct his own defense (a) The standard for competency to stand trial and competency to represent oneself is different (b) When mentally incompetent represents herself, it does not comport with dignity e. Due to higher standard in Edwards compared to Godinez, it is possible for an individual who is not competent to stand trial without a lawyer to be competent plead guilty without a lawyer f. State may appoint standby counsel (McKaskle) (i) Right to defend self is violated if counsel: (a) Makes or substantially interferes with any significant tactical decision, controls questioning of witnesses, or speaks instead of the on any matter of importance (actual control) -OR34

(b) Destroys jurys perception that is representing himself Counsels participation and consultation must typically be outside the presence of the jury (ii) No constitutional right to standby counsel or hybrid representation (iii) Once D invites or agrees to substantial participation by counsel, subsequent appearances must be presumed to be with acquiescence (a) Until expressly and unambiguously renews requires for silence by standby (iv) Generally no ineffective assistance of standby counsel claims unless erroneous legal advice g. Some states have appointed where representation is absolutely necessary to ensure a fair trial: physical disability/speech impediment or educational deficiency (Pickens) h. No right to self-representation on appeal (Martinez) F. Right to Effective Assistance 1. Retained Attorneys (Cuyler) a. Defendants who retain their own lawyers are entitled to the same protection as defendants for whom the State appoints counsel b. States conduct of a criminal trial itself implicates the State in the s conviction, thus no basis for drawing distinction that would deny equal justice to s who want their own lawyer 2. 6th Amendment Violations
Types Outright Denial (Gideon) Constructive Denial Interference with Relationship (i.e., judge not allowing to speak with counsel to prevent coaching prior to testifying) Denial at Preliminary Hearing Test Prejudice Presumed (no impact on outcome has to be evident on record) Prejudice Presumed Prejudice Presumed

Relief unless Prosecutor can prove: Denial was harmless beyond a reasonable doubt (did not affect trial)

Deficiency in Performance (Strickland)

Relief only if can prove: 1) Attorneys performance was unreasonable, outside the rang of professional competence -AND2) Reasonable probability that but for the 35

attorneys error, the result of the proceeding would have been different Actual Conflict of Interest Relief only if can prove: 1) Counsel actively represented conflicting interests -AND2) Conflict adversely affected attorneys performance (not actual prejudice)

36

3. Actual or constructive denial of the assistance of counsel presumed to result in prejudice a. Remedy for any actual or constructive denial is new proceeding (Perry) 4. Interference a. Prejudice presumed b. Where trial court has prevented counsel from utilizing certain adversarial procedures 5. Denial at prelim a. Relief unless prosecutor can prove denial was harmless beyond a reasonable doubt (did not affect trial) 6. Ineffective Assistance of Counsel (Strickland) a. Performance Prong- Attorney was deficient, unreasonable, outside the range of professional competence (i) Action cannot be considered sound trial strategy (a) Not the result of a reasonable professional judgment (b) Strategic choices made after thorough investigation of law and facts are virtually unchallengeable (ii) Strong presumption of reasonableness that must be overcome (a) Attention to certain issues and not others reflects trial tactics rather than sheer neglect (Richter) (b) Reasonable miscalculation, lack of foresight or failing to prepare for remote possibility not enough (Richter) (iii) Objective (not subjective) standard (a) Could that have been reasonable not What did this attorney think (iv) Guides include prevailing norms of practice reflected in AMA standard and the like (compare Van Hook with Padilla & Frye) (a) National, not local standards b. Prejudice Prong- Reasonable probability that but for the attorneys error, the result of the proceeding would have been different (i) Undermine confidence in the outcome (ii) Totality of the evidence (weak evidence likely to be infected) (iii) Any adverse change in the outcome (a) Adverse impact = sentence, charge c. Standard (i) Reasonably effective assistance, based on the facts of the particular case viewed as of the time of counsels conduct (ii) Whether reasonable probability that absent the errors fact finder would have had a reasonable doubt respecting guilt d. Measuring Prejudice During Plea Negotiations (i) Challenged after pleading guilty (Counsel misunderstood terms; misled about consequences) (a) Reasonable probability that but for counsels error, would have insisted on going to trial (Hill) (b) Lower courts Post-Padilla
37

Must show rational reason he thinks he could be acquired or a reasonable probability of an outcome more favorable that what he received as part of his plea bargain

Reasonable probability that would have received more favorable deal (lesser charge and/or sentence) would have accepted, P would not have withdrawn and jury would have accepted (ii) Challenged after losing at trial (Counsel failed to communicate plea offer; counsel misrepresented case) (a) Reasonable probability would have taken the deal and not gone to trial (Lafler) (b) Remedy Resentencing Order Pros. to reoffer plea and IF accepts, decide whether to accept 7. Actual conflict of interest a. Prejudice presumed (Cuyler) because counsel breaches duty of loyalty b. Examples (i) Previous or simultaneous representation of co-Ds (a) Holloway; Sullivan (ii) Previous representation or current representation in another matter a victim (iii) To be called as a prosecution witness (iv) Professional or personal interest that conflicts with D (a) Royalty agreement (b) Potential criminal or civil liability (participant in crime) (c) Under investigation for crime by same prosecutors office (d) Romantic relationship with prosecutor, codefendant, witness, judge or juror (e) Applied for job with proseuctor (v) Fee arrangement creates conflict (a) Third party with interest in case paying counsels fee (vi) Counsel has obligation to deliver physical evidence to the police c. Judicial Duty to Inquire- Rule 44(c) (i) Judge must inquire about conflict whenever there is joint representation (a) Advise defendants of right to conflict free counsel (ii) If actual conflict found: (a) Secure separate counsel; or (b) Knowing and voluntary waiver of right to conflict-free counsel d. Review After Conviction (i) Automatic Reversal (a) Defense counsel objects to representing two or more defendants

(c) Frye

38

(b) IF judge failed to inquire into conflict, reversal is automatic without regard to prejudice (Holloway, discussed in Mickens) (ii) Defendant must show ADVERSE Effect on Representation (a) No objection by attorney (b) Conflict from multiple or sequential representation amount to denial and require relief if defendant can prove (Cuyler; Mickens) Counsel actively represented conflicting interest; and, Conflict adversely affected the attorneys performance Need not show reasonable probability of different outcome (c) No conflict if attorney disregarded interest and pursued defendants interest (iii) Strickland Standard (a) Other conflicts (not multiple or sequential) are evaluated under this standard by most lower courts (b) Problems Remedy comes too late for most to use In most states after direct appeal in postconviction relief Typically been 3 years and half of those who would claim IAC are out) Prosecutors trade concessions for waiver of right to attack with IAC in plea deals Removes incentive to challenge inadequate lawyering Conflict of interest since need advice of lawyer to make waiver and this would indemnify defense attorney When litigated on post conflict, many IAC claims litigated without counsel Requiring prejudice means even most proven error goes uncorrected Defense lawyers suffer no consequent when new trial/sentencing/appeal ordered

G. Reforming Indigent Defense 1. State structures a. Individually appointed private attorneys (i) Appointed by court through judicial discretion in appointments or neutral rotational system b. Public defender offices
39

c. Contract-attorney organization: contract firm agrees to cover indigent defense docket for a flat fee or hourly fee with caps d. State oversight systems: qualifications, caseload limits, attorney training; special standards for death penalty representation 2. Some rights should not be waivable a. Procedures provide little protection because they can be waived, reform must come through political reform 3. Protect on front end with Non-Waivable Controls (Proposed Reforms) a. Statewide rather than local funding/control (i) Eliminates differences b. Minimum funding mechanisms (i) Parity with prosecution c. Agency Oversight d. Incentives e. Secure more high quality attorneys (i) Repay student loans of graduates who become defenders and prosecutors f. Create national Center for Public Defense Services (i) Independent standards, best practices 4. Tension between Court mandated procedural rights and reality of funding, crime rates, and crime definitions (Stuntz) a. Underfunding, overcriminalization, and oversentencing have increased as procedure has expanded b. Underfunding of defense counsel limits the number of procedural claims that can be pressed (i) Where legislature disagrees with courts rules, underfunding reduces number of claims pressed (ii) Courts should set funding floors (iii) Defense counsel more likely to assert constitutional argument than to investigate factual issues c. Expanded liability definitions and higher sentences make investigations and plea bargains easier (i) Officers can arrest for a minor offense to search or question suspect on a major one (ii) Leaving trivial laws on the books allows prosecutors to push for pleas without proving elements of harder-to-prove offenses d. Countermajoritarian restraints on the criminal process can succeed only at a cost, and the cost is disproportionately imposed on those who least deserve to bear it (i) Costs borne by the poor: Because wealthier defendants cost more to prosecute (more money to press procedural issues), steer towards prosecution of indigent defendants e. Constitutionalizing substantive criminal law would not have these negative effects, but courts leave it to the legislature; courts should be more proactive in funding, definition of crime, and sentencing

40

(i) Courts motivated by desire not to trench legislatures territory and to stick to law-like and less contentious issues (ii) Allows state to end-run much criminal procedure (iii) System might be better off if Warren court had worried less about criminal procedure and more about criminal justice

41

VII.

Access to Evidence A. Introduction 1. Rationales a. Pro-discovery (i) Ds statement ensure accuracy (discover fabrications) and fairnrness (ii) Encourages settlement (iii) Avoid surprises at trial b. Anti discovery rationales (i) May lessen witness protection or protection for undercover informants (ii) Perjury concerns: alibi or defense to fit with Ps case? (iii) Compromises work product protections 2. Models a. FRCrP 16 narrowest model b. ABA 1st: Most states follow this in-between approach (i) Defense discovery equivalent to FRCrP 16 (ii) Prosecution standards roughly similar to Rule 16 + Rule 12.1 (notice of alibi) and Rule 12.2 (notice of an insanity defense c. ABA 3d: Broadest discovery d. Three aspects cause most significant variations (i) What is and isnt discoverable (ii) General/broad (but similar) language leads to varying interpretations (iii) Interaction between court inherent authority and statute or rule (a) Preemptive- Authority limited to identified, potentially with a catch-all provision (b) No set range of authority- Retains inherent right when to issue discovery (Federal) e. Defense discovery is uniformly broader than prosecution 3. General Scope a. Limit disclosure to items within the possession, custody or control of the particular party 4. Disclosure always made subject to possible restriction through judicial issuance of a protective order on sufficient showing (Rule 16)
Reasons to limit discovery of Pros. Info. -Witness Intimidation - Perjury (change story) -Evidence Tampering -Work Product Reasons to promote full disclosure of info. -Accuracy (truth) -Efficient (settlement) -Fairness (balance/notice) Reason to limit Pros. discovery of info. -5th Amendment Privilege against self incrim. -Work Product -Attorney client privilege -Due Process

42

B. Defendants Constitutional and Statutory Rights to Discover Government Evidence 1. Largely a matter to be determined by state legislative or judicial policy a. Free to set as widely or as narrowly as pleased, barring a limited number of const. exceptions decided by SCOTUS (i) State granting discovery to the pros. must provide at least reciprocal discovery to the defense (Wardius) (ii) Due process obligation to disclose to the defesne such evidence with the prosecutors possession as it is both exculpatory and material (Brady) 2. Purpose of discovery is to prevent surprise at trial, thus prosecution provides advanced notice of the evidence it tinted to produce at trial a. All jurisdictions also provide discovery of at least some intned within prosecutions control that the pros. does not intend to introduce at trial 3. Defendants Oral Statements, 16(a)(1)(A) a. Almost all states b. Rationale (i) Less worry about witness intimidation; (ii) D may need to prevent perjury or inaccuracy (iii) Encourages settlements c. Rule: (i) Upon Ds request (ii) Turn over substance of statement (iii) If relevant (protects against statements made in connection with other ongoing inquiries) (a) Broad view, as prosecutor cannot really tell what would be relevant to defense (iv) In response to interrogation by person knew was government agent (a) Protects against discovery of statements by undercover agents (v) If government intends to use the statement at trial (a) Avoid statements only used for impeachment and keeps prosecutors from having to seek out discoverable item (vi) And knows or should know through due diligence that the statement exists 4. Defendants Written or Recorded Statements, 16(a)(1)(B) a. All jurisdictions provide for discovery if within prosecutions possession or control b. D request for inspection, copying or photographing c. Relevant written or recorded statement if (i) In governments possession, control, or custody (ii) P knows or should know statement exists d. Portion of written record containing substance of oral statement if (i) Relevant and (ii) D made statement in response to interrogation by person D knew was government agent e. Ds recorded testimony before GJ (i) Regardless of whether P intends to use
43

f. If contained in another witnesss statement, controlled by rules about witnesses 5. Statements to Undercover Agents are not discoverable a. D doesnt know they are government agents b. Protects identity for ongoing investigations 6. Witness lists a. Generally not discoverable under FRCrP/state law unless strong showing of special need (Stroop): (i) Difficult to prepare defense (ii) Complex paper trial with multiple Ds (iii) Critical, dispositive evidence would flow from anecdotal testimony or of other unindicted coconspirators (iv) No indication that D has past criminal record or involvement in violence or threat of violence (v) P does not indicate that providing witness name will make witness less likely to show at trial b. Some states, ABA1st recommend prosecutor turn over witness list c. ABA 3d requires disclosure of persons known to have knowledge of relevant facts, regardless of whether they will be called as witnesses 7. Statements of Others a. Statement 3500(e) and FRCrP 26.2 (must be one of the following) (i) Written statement made and signed by witness, or otherwise adopted (ii) Substantially verbatim, contemporaneously recorded recital of oral statement contained in recording or transcript of recording (a) Notes on statements was not a recorded statement because it was not recorded contemporaneously (b) But notes about a statement (including statements contained in police and investigative reports) count under ABA 3d rules (iii) Statement to GJ or transcript (iv) Applies to witnesss plea agreements b. Jenks Act (bars judge from ordering disclosure of Ws statements before direct testimony) (i) Time restriction rarely enforced as routinely ordered before trial (practice has departed from law) (ii) Law only requires disclosure if: (a) In possession of government (b) Defense requests (c) Material relates to subject matter of Ws direct testimony (d) Statement is one of listed above c. Pre-trial, no requirement to turn over witness statements, 16(a)(2) (i) Could lead to witness intimidation (ii) Time limit is rarely enforced: statements are routinely ordered before trial; judge may tell jurors that recess is due to prosecutions refusal to turn over statements before trial d. Some states provide for discovery of written, recorded, or oral statements of co-Ds (ABA 3d)

44

(i) States following ABA 1st only require co-D statements where co-Ds will be jointly tried (ii) States differ as to whether only statements intended to be used at trial are covered e. After witness testifies on direct, Jencks Act (i) D moves and court orders P to turn over any prior statements by the witness that relate to testimony that are in Ps custody f. P may request en camera review is government claims that statement does not relate to testimony g. Court may allow recess to allow D to review h. No access to statements by deceased witnesses 8. Ds Prior Record, 16(a)(1)(D) discoverable upon Ds request if in possession, custody, or control or if P should know record exists 9. Documents and Objects, 16(a)(1)(E) a. Photo books, papers, documents, data, photographs, tangible objects, buildings, places (i) Does not include state/agency records b. Rule 16 steps (i) Upon Ds request (ii) In governments possession custody and control (iii) Material to preparing defense OR P intends to use item in case in chief OR object was obtained from or belongs to D (a) Some states only require if it pertains to the case 10. Expert Reports, Discoverable under 16(a)(1)(F) a. Rule 16 (i) Ds request (ii) Written summary of expert testimony including opinion, basis and reasons for opinions and qualifications (name) (iii) If P intends to use at trial b. Rationale (i) Once opinion formed, little chance of intimidation or influence (ii) Lessens disadvantage between Ps early and complete investigation and Ds late and limited one (iii) Opportunity to examine closely and potentially seek assistance of defense expert 11. Police reports a. Generally protected by work product, even though police is not an attorney; or specifically exempted or by internal memo exemption (i) Some rules (FL) provide for disclosure unless court protective order (a) No reason needed to encourage government to do their job, thus privilege is not warranted b. Under 16(a)(2) barred c. Rationale (i) Preserving confidentiality of police sources, continuing investigations, investigative tactics (ii) Avoiding defense misuse to build red herring defenses
45

(iii) Encouraging filing of complete reports for internal review process 12. Government reports memoranda or internal documents a. Generally not discoverable, FRCrP 16(a)(2) (i) Subjecting these to disclosure might encourage statements preparer to be less accurate b. Protected by work product protections unless party seeking discovery shows a substantial need for the time and inability to obtain without undue hardship (i) If turned over should be careful not to produce mental impressions, conclusions, opinions, or legal theories of an attorney (ii) No distinction between prosecution and defense in need for a work product doctrine (Nobles) (iii) ABA 3d provides absolute protection only for opinion work product of the legal staff (does not extend to police reports) (iv) c. ABA 3d provides for discovery of material and information in possession or control of members of the attorneys staff and any others who either regularly report to have reported on this particular case 13. Depositions a. Not typically a discovery method unless 15(a)(1) applies\ (i) P must bear cost for indigent; expensive and should be rarely used (ii) Concerned about protecting witnesses from confrontation by the D; unnecessary burden even if no fear (iii) D could have right to attend deposition, leads to security risks b. Rule (i) If D shows (1) need to preserve testimony OR (2) exceptional circumstances and interests of justice (a) Witness about to die, be deployed C. Governments Rights to Discover Defendant Evidence 1. Rationale a. Limiting defense discovery was specific to that situation of protecting witnesses. Still concerns about accuracy, efficiency, settlement, fairness 2. Limits a. Due Process requires reciprocal discovery between D and P (Wardius) (i) Matter of balance (a) If it is to be a one way street there must be a strong showing of state interest to the contrary (ii) Prosecution must be willing to give up what is being requested of D b. Work product (and Rule 16(b)(2)(A)) protects reports, memoranda, or other documents made by the D or Ds attorney or agent c. Four requirements to trigger self-incrimination bar: (Izazaga) (i) Incriminating (ii) Personal to the D (iii) Obtained by compulsion (iv) Testimonial or communicative in nature

46

3. Notice of Defenses a. 12 states require the D give pretrial notice of the intent to rely on defenses (self-defense, entrapment, duress, intoxication, authority) (i) Not bound to raise the listed defense at trial, and prosecution cannot use against the D his failure to do so b. Notice of alibi does not violate Ds 5th Amendment right not to be compelled to be a witness against himself (Williams) (i) Petitioner was only compelled to accelerate the timing of his disclosure, because he would have had to disclose at trial (a) Very liberal rights to discover by defense thus reason why it is okay in light of Wardius (ii) Also serves the function of avoiding mid-trial delay and disposing of some cases without trials (iii) Dissent (a) But then D cannot later abandon the alibi and gives the P too much info about people who may know about D, which leads to evidence (related or unrelated offenses), rebuttal, too much information about Ds trial strategy (Dissent, commentary) (iv) State responses have varied (a) Many states provide prosecution discovery of defenses to be raised, defense witnesses, written or recorded statements of prospective witnesses, documents and tangible items (v) Rule 12 requires reciprocal state discovery if D files notice of alibi (see below) 4. Witness Lists a. 25 states and ABA 3d provide for D disclosure of names and addresses of witnesses D intends to introduce at trial (i) Reasonably anticipates it is likely to call (a) Includes persons to be called only if pros. pursues certain line of evidence when reasonably predicited (ii) These states also require defense discovery of Ps witness lists b. FRCrP 16 does not provide for witness disclosure, except alibi witnesses in Rule 12 (i) Disclosure lessens Ps burden of proving its case-in-chief (Prudhomme, Cal.) (a) But later amendment to Cal state constitution led to new decision upholding reciprocal disclosure of the names and addresses of witnesses it intends to call at trial (Izazaga) c. Alibi Witnesses, Rule12.1: After P request, D must turn over name, address, and number of each alibi witness on whom the D intends to rely (i) But if D files alibi notice, P must disclose the name of each witness that the government intends to rely on to establish that the D was present at scene of the alleged offense and any rebuttal witness to Ds alibi defense 5. Witness Statements
47

a. States that require witness lists also require written or recorded statements of the listed witnesses when statement is in Ds possession (i) Does not violate 5th Amendment protection against self-incrimination (Nobles) (a) Compelled statements are those of third parties (b) D did not prepare the report and did not convey any information to the investigator (c) The fact that the investigator works for D is immaterial b. Mass. order requiring defense furnish statements by prosecution witnesses that D intended to use at trial upheld (Commonwealth v. Durham) (i) Did not violate constitutional confrontation right (ii) Dissent argued that it impaired cross-exam by giving prosecution advance notice of inconsistencies c. Rule 16(b)(2)(A) prohibits discovery of statements made to D or Ds attorney if made by D, government or defense witness, or prospective government or defense witness 6. Documents and Tangible Objects a. FRCrP and both ABA standards provide for court-ordered defense disclosure of documents and tangible items the defense intends to use at trial 7. Scientific reports a. Rule 16(b)(1)(B) requires disclosure if (i) D requests (ii) Item is within Ds possession, custody, or control, AND (iii) D intends to use the item or call the witness at trial b. Limits (i) Due Process requires reciprocity (ii) 5th does not apply because D himself is not being asked to testify (Nobles) (iii) But Nevada has found that it does violate self-incrimination because D is forced to disclose information that he never intended to disclose at trial, some of which could be incriminating (Binegar) (iv) NJ found that forcing report of experts violates right to counsel because D must be able to rely on confidentiality in seeking an experts advice (Mingo) (a) Other states have rejected (v) Work Product (a) Waived if intend to present expert report/testimony at trial (b) If does not might protect (court split) Can be abrogated by statute or not protected by state Rarely been held that requiring disclosure if a violation (vi) Attorney-Client privilege could bar (p. 1227) in some states if attorney or client discloses info for purposes of trial prep (a) Other states reject because expert is not attorney; some allow privilege if client makes statements to the expert or attorney reveals private info received from the client

48

c. Easiest case for discovery because government will need to consult its own experts D. Remedies for Discovery Violations 1. Violations by the Prosecution a. Range of remedies (i) Ordering immediate disclosure (ii) Granting a continuance (iii) Excluding evidence: but this could harm accuracy and courts are hesitant (iv) Charge directing the jury to assume certain facts that might have been established by nondisclosed evidence (v) Contempt (vi) Mistrial: but there must be manifest necessity or new trial will be barred by DJ (vii) Dismissal of the prosecution (viii) Excluding testimony of unlisted witnesses (alibi and insanity notice statutes) except for good cause shown b. Selecting a remedy (i) Consider: Culpability, prejudice, lowest risk to accuracy (ii) Preference for continuance where party responsible for violation acted in good faith; court should determine if potential prejudice can be alleviated by continuance (a) But if length of needed continuance would be too disruptive or continuance wont respond to prejudice (after opposing side has committed itself to an inconsistent position), may need to consider other remedies: exclusion, jury instruction, mistrial (iii) Late disclosures that contradict a misleading defense have led to contradicting remedies, depending on the court (a) Letter that undercut Ds claim of good character and noninvolvement in the crime: court found that D had knowledge of letters existence (he had written it) and therefore there was no prejudice (People v. Taylor, Mich.) (b) Testimony that undercut Ds Costa Rica alibi: attacked very foundation of the defense strategy and so serious a detriment to preparation should lead to a new trial (Noe, 11th Cir) 2. Violations by the Defendant a. Remedies: Exclusion? (i) Exclusion would implicate Ds right to present a defense and right to the compulsory process (ii) But excluding testimony of a favorable witness is not absolutely prohibited by constitutional right to use of compulsory process. Pattern of discovery violations may create suspicions that violations were designed to present fabricated testimony, and appropriate to exclude (Taylor v. Ill)

49

(a) If omission was willful and motivated by a desire to obtain a tactical advantage and minimize effectiveness of cross and ability to get rebuttal evidence, appropriate to exclude (b) Integrity of judicial process at stake; D is penalized for attorneys conduct (Dissent disagreed) (iii) Allowing a surprise witness would lead to delay, rebuttal witnesses; would not cure prejudice because P would have to recall earlier witness to rebut and would seem surprised and exclusion was proper (Tyson v. Trigg) b. Generally may not exclude Ds testimony (Rule 12.1(e) (i) Does right to testify require greater constitutional protection than right to present witnesses; is concern less important since state has to prove Ds presence (ii) Some states have barred Ds from giving alibi testimony beyond denial of presence at the scene when no alibi notice was provided E. Governments Constitutional Duty 1. To Disclose Evidence (Brady Rule) a. Prosecutors has a constitutional duty to disclose exculpatory evidence that is reasonably likely to affect the outcome at trial or undermine confidence in the result (Brady; Bagley) (i) Test (a) Favorable to the accused Substantive and impeachment treated the same (b) Material to guilt or punishment ABA would have P turn over all evidence Four Guiding Principles (Whitley) Does not require demonstration by a preponderance that disclosure would have resulted in acquittal (matter of fair trial, where verdict is worthy of confidence) Not sufficiency of evidence test (not necessary to show that would not have been a conviction) Once const. error found no need for further harmless-error review Judged by reference to suppressed evidence considered collectively, not item by item (cumulative effect of suppression) (ii) Admissibility of Material Requested (a) Not inadmissible information (b) But a majority of states require disclosure if it could lead to admissible evidence
50

(iii) Defense Diligence (a) Some courts have held D must is responsible for failure to request known items based on unknown to the defense language Leaves pros. to assume D has not interest in item despite potentially exculpatory character (b) Requires showing D was aware of the potentially exculpatory nature of the evidence as well as it existence (iv) Pros. or Agents Possession or Control (a) Lower courts extend to material within files of various investigative agencies participating in case (b) Fed. Agency consider: Whether played an active role Degree to which information gathered by the pros. has been shared with the agency (c) Burden is on the pros. to ensure communication of all relevant information on each case (v) Known to P or agents working on the case (a) ABA would include investigative team (b) P not required to conduct investigation for evidence though (vi) Not disclosed (a) If D had other access to info, courts may not find a violation b. Violations (i) Only requires that turning over those files which would deprive D of a fair trial if suppressed (a) Exculpatory evidence (Brady) (b) Impeachment evidence (Bagley) (ii) Use of perjured testimony/failure to disclose that testimony used to convict was false (Mooney/Napue) (a) P must show error was harmless (iii) Strickland Formulation- Covers failure to respond to general request or specific request, and no request (Agurs) (a) D must show (1) Error; and, (2) Reasonable probability that, had the evidence been disclosed, the result of the proceeding would have been different (sufficient to undermine confidence in the outcome) (iv) Once constitutional error is found, no need for harmless error analysis (Kyles v. Whitley) (v) Court can consider whether failure to respond to a specific request had adverse effect on preparation or presentation of defense c. Timing (i) Routinely not an issue as most prosecutors will disclose any Brady material upon request (ii) In time for effective use at trial
51

(a) Scientific evidence must be disclosed sooner (iii) When evidence is covered by Jencks and Brady (prior recorded statements of a prospective government witness), courts are divided on timing (iv) Disclosure of impeachment material need not be made in time to consider before plea agreement (Ruiz) (a) Impeachment is about fairness at trial (b) Plea only requires understanding nature of right, not detailed consequences of invoking (c) Could interfere with pleas d. ABA Model Rules 3.8(d)- Make timely disclosure to the defense of all evidence or information known to the pros. that tends to negate the guilt or the accused or mitigates the offenses (i) Argues that material should be turned over in time to be used in deciding whether to plea (ii) Not limited to admissible evidence (iii) More demanding and separate from const. case law (a) Does not regard anticipated impact on trial outcome (iv) Consent of D does not absolve pros. of duty imposed 2. To Allow Access To Evidence a. To witnesses (i) Interference with Access (Valenzuela-Bernal) (a) Deportation of witnesses violates compulsory and/or due process if D can show that evidence lost would be both material and favorable to the defense; sanctions warranted only if there is a reasonable likelihood that testimony could have affected the trier of fact Brady applies but deportation alone is not enough to establish a violation Government has manifold responsibilities in immigration cases (ii) Intimidation/Threatening off Stand (Webb) (a) State (judge or prosecutor) may not impose pressure on witness not to testify Witness must be important to the defense, material D was denied witnesss testimony as a result of the prosecutions action (threats to prosecute for perjury) Prosecutor should convey perjury information through witnesss counsel, not personally Recent Broadcom case dismissed indictment where prosecutor called witness and threatened to prosecute for perjury if testimony differed (iii) Immunity

52

(a) No general const. right to immunity for defenses witnesses testifying on his behalf has been found in the 6th Amendment compulsory process clause (Turkish) Difficult to see how the [compulsory process clause] of its own force places upon either the pros. or the court an affirmative obligation of replacing the protection of the self-incrimination privilege with a grant of use immunity (b) Court can grant only in egregious cases; immunity is a charging decision (DAntonio) (c) Must find: (Bahadar) P discriminatorily granted immunity to its own witnesses: does it look like prosecution is game-playing with grants Witness has material, exculpatory, noncumulative evidence Information is unobtainable from other sources b. Critical Informants (Rovario) (i) Informers privilege may give way to need for a fair trial where (a) Is the informant the sole participant in the transaction and central to the events? (b) Does the witness have exculpatory evidence (Brady applies) (ii) Balance need to reveal identity (a) Crime charged, possible defense, significance of the informers testimony, and other relevant factors 3. To Preserve Evidence a. Preservation/Destruction of Physical Evidence (i) If destruction of evidence was done in bad faith, violation of due process (Youngblood) (a) In light of procedures used whether chances that preserved samples would have exculpated the D, even if might have shown inaccuracy, is there an alternative means of demonstrating their innocence (Trombetta) (b) If state has a destruction policy that it followed, it is not destroying evidence in order to win (c) But many states have adopted a multi-factored balancing test: is this the kind of evidence that would exonerate? How culpable was the state? (ii) Was later exonerated, though still good law (a) Many jurisdictions have backed off of this standard b. Sanctions (i) Loss of conviction (ii) Contempt (iii) Disciplinary sanctions from violating Rule 3.8 or use of false evidence

53

VIII.

Pleas and Bargaining A. Types of Plea Agreements 1. Sentence Recommendation a. Pros. promises that will recommend to the court: (i) A sentence favorable to the D (ii) Will not seek the maximum penalty; or (iii) Will refrain from making any recommendations b. Promise to suggest to the trial judge a mutually satisfactory term of years (i) MUST be reasonable expectation of judicial acceptance of recommendation for it to be valuable to D 2. Plea to a Lesser Included Charge a. Pros. recommends court accepts lesser offence included in offense actually charged: (i) Court permission is usually required (ii) In some jurisdictions, Pros. must file reasons for recommending 3. Offer to Dismiss Certain Criminal Allegations a. Court approval usually required b. Apparently advantageous bargain may actually not be (i) Tendency of courts to sentence concurrently or to suspend sentence on all but one or two of the multiple charges B. Rule 11 1. Plea agreement may specify that government will a. Not bring or will move to dismiss, other charges (i) May also charge with lesser included offense (ii) Requires leave of the court to dismiss (FRCrP 48(a)) b. Nonbinding recommendation or not oppose Ds request that a particular sentence or range is appropriate or that particular provision or sentencing factor does or does not apply (i) D may not withdraw the plea if the court does not follow the recommendation or request; always a risk c. Agree to specific sentence or sentencing range: binding if the court accepts C. ABA Standards 14-1.8: Appropriate for the court to approve sentence concessions where 1. D expresses genuine contrition and shows a willingness to assume responsibility for his or her conduct 2. Concessions will make alternative correctional measures available where these will be effective 3. D has demonstrated remorse or consideration for victims in foregoing public trial 4. D has given or agreed to give cooperation to apprehend equally or more serious criminals D. Incentives to Support Bargaining 1. Prosecutor a. Administrative efficiency (i) Alleviates overcrowded dockets b. Using for weak cases likely to lose at trial c. Where D can cooperate and help convict others d. Spares V from trial
54

e. Punishment starts sooner, increased time for rehabilitation f. Acceptance of responsibility allows for remorse g. Avoid dangers of pre-trial release h. D can waive pre-plea errors 2. Defendants a. Leniency (i) Allow prosecutors to manipulate charge and courts to manipulate sentences for less culpable offenders b. Expression of remorse c. Avoid uncertainty/risk of trial (i) Avoid cost of gambling and losing (Scott) d. Negotiate for other concession (i) Assets, protection, dont charge my mother 3. Judges a. Clear docket/turn cases over faster E. Problems 1. Many plea agreements require D to waive appeal rights a. Waiving future risk is uninformed b. Even waives serious error c. Although many courts would find an exception for actual innocence because of change in substantive law (Bauza) 2. May scare innocent people into pleading a. But often its a question of value judgments about conduct: we simply choose to criminalize or not criminalize some conduct and there is no objective truth about whether it was criminal or not (Enker) b. If innocent person receives a giant discount for pleading, does this start to look like coercion? (Schulhofer) 3. Privatizes public disputes a. Destroys the idea of objective societal determination of moral guilt 4. Many Vs want more punishment 5. Dilutes moral force of criminal sanction and distorts public perception and criminal legislation (Arenella) 6. Sentencing disparity between those who plea and those who exercise right to go to trial. But: a. 1st Cir. has held that sentencing disparity does not unfairly burden the right to go to trial (Rodriguez) b. Proper for the court to consider the Ds whole person and personality for whatever light those may shed on the sentencing decision (Grayson) c. Does putting victim through trial, eliciting details of the crime merit a higher sentence? d. Rewarding person for positive benefits of apprehending others

55

F. Due Process: Intelligent and Voluntary 1. When are Waivers Involved Voluntary? a. Voluntary Charging Incentives (i) Pros. may threaten to add charges if D doesnt take a plea (Hayes) (a) Prosecutor could have brought either charge from the beginning and substituting the higher charge after D refuses to plea isnt punishment (b) D simply chose to forego leniency (c) Taking a plea would have allowed him to avoid the cost of gambling and losing at trial; he may be in the same position as D who goes to trial and loses (Scott) (d) A contrary rule would incentivize prosecutors to always bring the higher charge in the beginning (ii) Pros. agrees to drop death penalty if D pleads guilty (Brady) (iii) Pros agrees to drop charges against D1 on condition that D2 first pleads guilty (Lafler) (a) Courts will look closely to determine if involuntary b. Voluntary Sentencing Incentives (Prosecutor) (i) Much greater plea if go to court (multiple times larger) (ii) Downward departure from the presumptive range (iii) Dropping sentencing enhancements (iv) Foregoing mandatory minimum sentence options (v) Making use of discretionary, alternative sentences (a) i.e., Treatment programs, suspended sentences (vi) Stipulation to criminal history or other facts (vii) Stipulating to sentence cap c. Sentencing Incentives- Judicial Involvement (i) Post-Trial (a) Heftier sentence given by judge than offered by Pros. Although it is judge saying you dont get consideration you would get if you pled guilty, it looks vindictive (b) Courts find non-vindictive reasons for upholding Large amount of knowledge (ii) Pre-Trial during Plea Negotiations (a) Could render plea involuntary (b) Could be barred by statute/rule/case law (c) More coercive if: Judge initiated Statements were to D not defense counsel only Sentence predictions are certain, not merely possible If claim will be convicted Short amount of time between involvement and plea Same judge at time of plea (d) Why limit?

56

Create impression in mind of D that will not receive fair trial before this judge Difficult for judge objectively to determine voluntariness of plea when offered To extent of promising certain sentence, inconsistent with theory behind use of presentence investigation report Risk of not going along with disposition desired by judge may seem so great to the D that will be induced to plead guilty even if innocent d. Statutory Sentencing Incentives (i) Statute codifying higher maximum sentence for Ds who exercise right to trial (a) If D goes to trial and receives sentence higher that that authorized had he pleaded guilty it is PROBABLY a violation of Due Process Statute doing this with death penalty struck down (Jackson) No problem when codified difference between available MINIMUM sentence for plea and trial (non-capital offense) but maximum the same (Corbitt) (b) If D pleads guilty to avoid it is NOT involuntary (Brady) e. May Be Involuntary (i) Induced by: (Shelton) (a) Threats (or promises to discounting improper harassment) (b) Misrepresentation (including unfulfilled or unfulfillable promises) (c) Or perhaps by promises that are by their nature improper as having no proper relationship to the pros. business (bribes) (ii) Statutory scheme where highest penalty is reserved exclusively for those who opt for trial (Jackson) (iii) Plea made on constitutionally defective advice from counsel (Bradshaw v. Stumpf) (iv) Could not have understood the terms of the bargain he and the state agreed to (Bradshaw v. Stumpf) 2. When are they Knowing or Intelligent? a. D must understand: (i) True nature of charges (Brady) (a) Includes knowledge of the critical elements (intent in 2nd degree murder) (Henderson) Non-obvious and those which make a difference between conviction offense and lesser offense May include meaning of attempt (ii) Type and maximum sentence, but not collateral consequences (a) Actual sentencing possibilities (Williams)
57

If not question of whether accurate info would have made a difference in plea acceptance (b) Does not require: Understanding parole eligibility Recidivist penalties for later crime Inform that conviction may mean deportation, loss of right to vote, etc. Knowledge of rehabilitation programs available (iii) Generally what rights waiving (a) Need not be told specifically by judge (Wilkins) (b) More may be required by statute/rule (iv) In practice judge and pros. tell everything in attempt to bulletproof the record b. Advised by competent counsel (Brady) (i) Where D can show counsel was ineffective and, but for bad advice, would have refused plea and gone to trial (Hill) c. Nothing to indicate that he was incompetent or otherwise not in control of his mental faculties (Brady) d. D does not have to understand the strength/weakness of evidence of guilt (i) Brady material not constitutionally required to be disclosed in time for plea (Ruiz) 3. Reviewed for Harmless Error (Bousely) a. If judge misses a critical a critical warning or admonishment, relief unless pros. can show beyond a reasonable doubt that either: (i) D actually understood (e.g. counsel (Stumpf) or pros. explained); or, (ii) If D did not understand, had he known he would have pled guilty anyway b. If Rule 11 violated: (i) Either D knew information anyway; or (ii) D would have pleaded guilty anyway had he known G. Breach and Remedies for Breach 1. Breach by Prosecution a. When a plea rests in any significant degree on a promise or agreement of the prosecutor, so that it is part of inducement or consideration, violation of that promise or induce is breach (Santobello) (i) Want deals to be negotiated (a) If D does not believe that they will get deal they were promised unlikely to make them b. Withdrawal (i) When a prosecutor makes a bargain, he can withdraw before it is accepted by the judge, unless the D has detrimentally relied on the agreement (Mabry). (ii) Detrimental reliance includes (a) Providing info to the government (b) Testifying in court
58

(c) Confessing guilt (d) Returning stolen property (e) Monetary restitution (f) Failing to file a motion to have charges presented to a GJ (g) Submitting to a lie detector test (h) Waiving procedural guarantees c. Things that arent breach (i) Not breach where court considers other conduct relevant to sentencing (US Sentencing Guidelines) d. Remedy (i) Options (a) Withdraw the guilty plea and deal with original charges (trial/new negotiations); OR (b) Demand specific performance of the agreements (ii) Defense may prefer one over the other due to change in knowledge over the case or the surrounding circumstances (iii) In most jurisdictions, it is up the to the JUDGE to determine which option (Santobello) (a) Remanded to lower court for consideration of which remedy J. Douglas thought weight should be accorded to Ds preference of remedies J. Marshall and dissent thought plea should be vacated 2. Breach by Defendant a. Before the court accepts the plea (i) D may withdraw for any reason or no reason (FRCrP 11(d)(1)) b. Before sentencing (after acceptance) (i) D may usually withdraw plea with good cause (FRCrP 11(d)(2)) (a) Fair and just reason c. After sentencing (i) Cannot withdraw unless: (a) Manifest injustice (b) Plea conditioned on certain sentence and that sentence not imposed Agreement included a term binding the judges discretion in sentencing and the limitation was accepted by the judge (as in a (c) plea), but the sentence imposed violated that limitation d. Remedy (i) Where D refuses to testify (even for second or successive trials), state can move forward as though no agreement had been reached (Ricketts) (a) D should make sure there is no clause that says P is sole decision-maker on whether breach occurred

59

H. Judicial Regulation- Policing the Bargain 1. Rule 11 a. Before accepting a plea, court should inform D of right to plead not guilt, right to jury trial, right to counsel, confrontation right, waiver of rights, nature of the charge, maximum possible penalty including imprisonment, fine, and term of supervised release, mandatory minimum penalty, applicable forfeiture, restitution, special assessment b. Ensure that plea is voluntary and did not result from force, threats, or promises c. Ensure factual basis for the plea 2. Potential grounds for Judicial Refusal a. Agreement not voluntary or knowing (coerced or unintelligent) (i) D must understand the charge, including critical elements (Henderson) (ii) D must understand the max sentence, but not collateral consequences (parole, mandatory minimum, recidivist penalties for later crimes, SORNA, restitution, or forfeiture) (iii) Prosecutor cant make a misrepresentation, use perjured testimony b. Disagrees with Charge (i) Very little power to second guess (a) Separation of powers requires that judges not interfere with charging decisions (Newman) (ii) Too Harsh? (a) No (Newman) Not the role of judges to ensure that plea agreements are fair among Ds (iii) Prosecutor refused to charge bargain? (a) Does reason matter? YES- Race or gender, vindictiveness MAYBE- Dislike of Ds attorney, if no justifiable law enforcement reason NO- Victim wishes (improper delegation) All states have some form of statutory crime victims bill, typically stating that able to communicate view on deal to pros./judge prior to offer/acceptance (iv) Too Lenient? (a) Yes, minority/some courts No right to plead guilty, even as charged Must provide individualized findings as to why lesser charge improper (b) No, most courts Violation of separation of power for Judge to assume prosecutorial role (In re US) No authority under FRCrP 48(a) to deny motion to dismiss
60

Unless government harasses D by repeatedly bringing charges and dismissing them (In re US) c. Disagrees with Particular Term (i) Bargained Sentence (a) May reject as too lenient or harsh (Ellis) Under Rule 11 must give D opportunity to withdraw (b) Police at plea hearing by either rejecting or accepting offer (ii) Illegal Sentencing Terms (a) Judge may reject Most Invalid, back to square one Some

If voluntary and knowingly agreed, ok (b) D generally will not be able to excise ONLY the illegal term and keep the benefits of the bargain i.e.- Coerced contributions, deprivation of certain rights, scarlet letter punishments, surrender of profits, banishments, military service (iii) Waiver of rights (a) Appeals Courts are divided Federal courts generally think trial judges abuse discretion to reject plea on this basis d. No factual basis for conviction (i) Court may accept guilty plea even if D insists on innocence (Alford) (a) But judge may refused to accept; some states ban (ii) Constitution only requires factual basis for Alford pleas (contests guilt) (a) BUT state statutes and Rule 11 require factual basis for all pleas (iii) FRCrP 11(b)(3) (a) Court shall satisfy itselfthat the conduct which the D admits constitutes the offense charge or an offence included therein to which the D has pleaded guilty (b) Protects a D who is in position of pleading (voluntarily with an understanding of the nature of the charge) without realizing that his conduct does not actually fall within charge

61

IX.

Trial by Jury A. Scope of the Right to Jury 1. 6th Amendment right to trail by an impartial jury is part of due process guaranteed to state Ds by the 14th Amendment (Dunacn) a. Rationale: (i) Prevent government oppression by providing additional alter of independent within judiciary (a) Corrupt or overzealous prosecutor; unfounded criminal charges could be brought to eliminate enemies (b) Judges could be too acquiescent to higher authority, biased, or eccentric (ii) Opportunity for society to participate in process 2. Waiver a. D has option of waiving a jury and having bench trial as it is principally for the benefit of the accused (Patton) (i) But FRCrP 23(a) requires (and Singer upheld) that waiver may be predicated upon pros. and judicial consent 3. For What Crimes? a. Certain petty crimes or offenses are not subject to the 6th Amendment b. All felonies and misdemeanors that carry a potential of more than 6 months are not petty(Baldwin) (i) Anything 6 months or less presumed a petty offense, but not automatically (Blanton) (a) Additional statutory penalties viewed in conjunction could rebut presumption if so severe they clearly reflect legislative determination that offense is a serious one Where legislature calls for $5000/individual or $10K for organizations, this could rebut (ii) Several misdemeanors or counts should not be joined in determining whether it is petty (Lewis) (a) Look to legislative intent to determine seriousness Fact that charged with multiple does not change legislative judgment as to gravity of a particular offense c. Depends on legislatures assessment of seriousness (i) Compare to right to counsel which requires actual incarceration and depends on judges assessment of seriousness 4. Sentencing a. D not entitled to sentencing by jury (Spaziano) b. Unless statute requires finding of aggravating facts (capital Ring) (non-capital statute, Blakely) 5. Size a. Six-person jury does not violate 6th Amendment rights (Williams v. Florida) (i) But 5 is too few: less likely to foster effective group deliberation, disparate verdicts, no meaningful minority group representation (Ballew v. Georgia)
62

6. Unanimity a. Twelve person need not be unanimous (Apodaca) (i) Lack of unanimity does not mean reasonable doubt (ii) 10-2 and 11-1 easily permitted, line is potentially 9-3 (75%) b. 6 person jury must be unanimous may be speculative at best (Burch) (i) Near uniform judgment of the Nation (a) Based on fact only 2 states allow non-unanimous verdicts by 6 member juries c. But most states require unanimity 7. Deadlocked Jury Instructions a. May give jury supplement instruction encouraging agreement b. Allen Instruction (i) Should examine question with candor and proper regard and deference to others opinions (ii) Duty to decide case if conscientiously can do so (iii) Listen with disposition to be convicted by others arguments (iv) If large number were for conviction, dissenting jurors should consider whether their doubt is reasonable (v) If large number were for acquittal, dissenting jurors should consider whether they might not reasonably doubt the correctness of judgment if it was not concurred in by the majority c. Many courts have adopted less coercive form of deadlock instructions than those in Allen (similar to those recommended by ABA) d. Judge may send jury back for additional deliberation even though indicated more than once they cannot agree or have requested to be discharged (i) Cannot be for unreasonable length of time or unreasonable intervals e. May abuse discretion in discharging jury too quickly, without manifest necessity (i) Will bar a second trial of the D unless he consented to discharge 8. Nullification (acquittal despite proof of guilt beyond a reasonable doubt) a. Court rules that may allow nullification (i) No directed verdicts (ii) Jury not required to explain acquittal (iii) No new trial after acquittal (DJ) b. Court rules that prevent nullification (i) Judge should not give formal jury instructions that jurors may nullify (Dougherty) (a) Allows them to act as mini-legisalture (ii) D not entitled to inform jurors (through argument or instruction) that a guilty verdict would carry a mandatory minimum sentence (PabonCruz, 2d Cir) (iii) Courts may approve of dismissal of juror who intends to nullify (Thomas, 2d Cir) (iv) Instruction that if jury finds all of the elemtns of the crime beyond a reasonable doubt, it must convict (Ragland) (v) Jurors take oath to follow the law
63

9. Legally Inconsistent Verdicts a. When multiples counts or Ds, jury will sometimes return verdict that are not logically consistent with one another (i) Ok for jury to convict on facilitation, but acquit on underlying felony (Powell) b. Not constitutionally intolerable and a D may challenge on appeal the sufficient of evidence supporting the guilty verdict (i) Upheld not just based on jury nullification power as Court has upheld inconsistent verdicts by State judges 10. Juror misconduct a. Forms (CRIMPROC 24.9) (i) Discussing the case outside of the jury deliberations (ii) Inspecting the scene of the crime (iii) Conducting experiments (iv) Using a dictionary to define a term in the judges charge (v) Lying during voir dire (vi) Considering outside information b. Must be raised before verdict (unless there is a non-juror witness) (Tanner) (i) Disrupt finality (ii) Full and frank discussion in the jury room, willingness to return an unpopular verdict, communitys trust in the system relies on secrecy (iii) Can question juror suitability during voir dire B. Selecting the Venire: Cross-Section Right 1. Basic Jury Selection Process a. Define Vicinage b. Select list(s) of residents c. Quality List (i) English speaking, non-felons, etc. d. Draw venire from qualified list e. Select jury from venire (voir dire) 2. Right to fair cross section is fundamental (Williams) a. Violated by systematic exclusion of women by requiring women to opt in to jury system (Williams) b. Only applies to venire; no requirement that petit juries must mirror the community (Williams) 3. Statutory/State Law Challenges a. Federal Jury Selection and Service Act of 1968 (i) Requires random selection (ii) No citizen shall be excluded from service on account of race, color, religion, sex, national origin, or economic status (iii) Procedure (a) Define vicinage (geographic area) (b) Select lists (c) Qualify lists: English-speaking, felonies, etc (d) Draw venire (e) Select petit jury from venire
64

4. Const. Challenges a. 6th Amendment Cross-section Violation (Taylor, Duren) (i) New trial required if: (a) D shows a cognizable group (Sufficiently numerous and distinct) Members share specific common characteristics (Barber) Underrepresentation by 18-34 year-olds not distinctive Amish comprise distinctive group when they make up 35% of community (Fulton) (b) Representation is not fair and reasonable in relation to the number of people in the community Absolute disparity analysis: difference of 10% between community and groups share of the venire (Mosley) Where group makes up less than 10% of population, use comparative disparity: divide absolute disparity by population figure for a group; 40% is borderline (c) Due to systematic exclusion by jury selection process Broad discretion granted to State in prescribe relevant qualifications and to provide reasonable exemptions Using voting registry (even where large number of Asians choose not to register) not systematic exclusion (Le) Disparity attributable to economic, cultural, social, or language considerations must be deemed unavoidable (Morales) -AND(d) State unable to show that selection mechanism advances a significant state interest Greene: exclusion of those charged with crimes Chidester: Farmers excluded during planting season b. Equal Protection Challenge (Castaneda) (i) D must persuade judge that government INTENDED TO discriminate (a) Usually accomplished if Shows prima facie case Statistical under-representation of suspect class Opportunity to discriminate State has no race-neutral explanation (ii) Displaced by easier-to-prove cross-section challenge

65

C. Selecting the Jury from the Venire 1. Access to Juror Information a. Most courts have rules limiting the use of an anonymous jury where names, addresses and sometimes places of employment are withheld from parties and press throughout (US v. Thomas) 2. Sources of Juror Information a. Questionnaires (i) Court uses detailed questionnaire to determine background and attitudes of prospective jurors b. Information pros. assembled from police records or past trials (i) Almost uniformly held that needed not reveal to the D information gleaned in this manner 3. Questioning the Jurors a. No constitutional right to have questions asked of jurors (Ristaino) b. But during capital sentencing of interracial crime, right to ask about racial prejudice (Turner) 4. For Cause Challenges (6th Amendment impartial jury right of the accused) a. Standard (i) Juror has state of mind which will prevent him from being impartial or from applying the law (a) Established either by direct admission, or implies b. Express/Admitted bias (i) Person is unable to impose the death penalty and views are unmistakably clear (Witherspoon). (a) Only a juror who is substantially impaired in his or her ability to impose the death penalty may be removed for cause. (Uttecht). (b) Reverse challenge by D were appears would automatically vote for death penalty in every capital case (Morgan) (ii) Could not convict 18 year old of statutory rape of consenting 15 year old g/f (iii) Statement that thinks the defendant is guilty (iv) Was victim of same sort of crime and admits that cannot apply presumption of innocence (v) Admits would be unable to believe anything a snitch/police officer says c. Implied bias (Juror says can be impartial but court rejects) (i) Fear that juror would use outside knowledge (a) Bias implied from extreme publicity (Irvin) (ii) Trial judge left with definite impression that potential juror would be unable to faithfully and impartially apply the law (inability to impose death penalty) (Witt) (iii) Association alone not enough (Salamone) (a) Membership is not a proxy for beliefs (iv) Not usually challengeable solely because of employment with the government (Dennis)
66

(a) But employment at a bank that was robbed may be enough (Allsup) d. Appealing erroneous for cause rulings: (i) Grants of cause challenges (knocking off someone who would have been impartial) not a basis for relief if you end up with an impartial jury (ii) Erroneous denials may be a problem if that person ends up on the jury; but if you use peremptory challenge on that person, cant claim violation for being forced to use peremptory (Ross) (iii) But the prosecution cant argue that Witherspoon violation is harmless because he had a peremptory challenge; would insulate jury selection error from meaningful appellate review 5. Peremptory Challenges (14th Amendment Equal Protection Rights of the Jurors) a. Violation if D can show intentional discrimination in his case (Batson) (i) Rejection of former rule in Swain that held must prove all cases b. Three Step Analysis (Purkett) (i) Prima facie showing of intentional discrimination or deliberate exclusion on the basis of a protected class (race/gender/ethnicity) (a) Narrower than 6th Am challenge which allows showing a pattern of discrimination against any distinctive group (b) Cross-section 6th Amendment challenge not applicable to selection from the venire (Holland) (c) White D is allowed to object to exclusion of black jurors to enforce jurors rights (Powers) Because injury is to excluded juror who is unlikely to challenge because of small damage awards and D shared motivation in the outcome (d) State can object to discriminatory practices by the D (McCollum.) (ii) Party defending challenge must present a group-neutral reason (Hernandez) (a) For excluding women, need exceedingly persuasive justification (JEB) Extended Batson protected class to gender (iii) Judge decides if party objecting has shown intentional discrimination by a preponderance (Miller-El) (a) Burden on challenging party (b) Divide between ethical obligation not to lie and courts potentially finding attorneys explanation incredible (c) If offered reason to for striking a black panelist applies just as well to an otherwise-similar nonblack who is permitted to serve, that is evidence tending prove purposeful discrimination (Miller-El) c. If court finds violation, must start with a new venire (i) Substituting someone of the same race doesnt vindicate excluded persons rights; compounds the discrimination
67

6. Eliminating Peremptory Strikes a. Should they be eliminated? (i) End racial the discrimination that they inject (ii) Litigants can misuse if they strikes fall below the threshold (iii) Asks court to second-guess stated reasons (ethical implications?) 7. Change of Venue a. Presumed pervasive prejudices mandating will only be presumed in the extreme case (Skilling) (i) Not where in large city, publicity not sufficient, no confession, cooling time before trial, mixed verdict D. Selected Trial Rights 1. Presence a. Rationale (i) 6th Amendment right to presence at trial is not an absolute right b. Two Prong Test (Stincer) (i) Confrontation (a) Does exclusion interfere with opportunity to effectively crossexamine? (ii) Due Process (a) Would present of the D at the proceedings have either: Helped D to defend himself; or Resulted in a more reliable/accurate outcome for that proceeding c. D may lose constitutional right to presence if: (i) After warning, D continues disorderly, disruptive, disrespectful conduct and there no effective less-restrictive means (Allen) (a) There is nothing constitutionally impermissible in continuing the trial in Ds absence (b) Binding or gagging the D is not fair or dignified (c) Holding D in contempt not sufficient (d) If D is expelled, court should try to mitigate the disadvantage by making reasonable efforts allow attorney-client communication and keeping D apprised of the progress at trial (ii) Failing to appear when at liberty and has duty to (Taylor v. US) (a) Court may begin a trial if D absent, only if D knew of the trial date and intentionally stayed away (Jefferson v. State) Lower courts found constitutional to begin without D (b) FRCrP 43 prohibits the trial in abstentia of a D who is not present at the beginning of trial (Crosby v. US) d. Which proceedings? (i) D only has a right to be present at trial and proceedings that could interfere with Ds opportunity to cross-examine a witness. Or any stage that is critical to the trials outcome or if his presence would contribute to the fairness of the procedure. (Stincer). (a) Due Process and 6th Amendment confrontation right

68

(ii) No right to be present at pre-trial competency hearing of minor victims: the children later testified in open court where D had opportunity to cross 2. Restraints a. Court may not use visible shackles to restrain D during trial (Deck) (i) Problems: (a) Undermines presumption of innocence (b) Interferes with right to communicate with counsel (c) Weakens dignity of court proceedings (ii) May only shackle in presence of a special need specific to the particular case, determined by the court b. Does not apply at time or arraignment or like proceedings before the judge (i) Meant to protect D from appearing before jury c. Having security in front row does not raise same problems 3. Confrontation a. The 6th Amendment bars the introduction of testimonial hearsay (Crawford) (i) Testimonial: (a) Prior testimony at prelim, before a grand jury or a former trial (b) Police interrogations (ii) Unless out of court statement was nontestimonial: (a) Made to police during an emergency (Davis) (b) Not in anticipation of litigation (iii) D has no opportunity to cross examine and witnesss out of court statements (iv) Unless the witness is unavailable at trial and D had a prior opportunity for cross-exam (Crawford) b. Look to primary purpose (Bryant) (i) Objective test (a) What wold reasonable people have believed was the prupose of the exchange under the circumstance? (ii) Factors (a) Where exchange occurred (b) Formality (planned/scripted) (c) Content of questions and answers (emergency or past fact) (d) Is question during or after the emergency (e) Medical condition of declarant (f) Nature of the threat (weapon/public) 4. Not to Testify a. 5th Amendment right to be free from compelled testimony b. Jury may not be instructed to infer and prosecutor may not argue guilt from Ds silence (Griffin) (i) There are a number of reasons the D may not testify (a) Timidity, nervousness, embarrassment, prior convictions (ii) Goes to harmless error (a) Unreliable inference and unwarranted penalty

69

c. d.

e.

f.

(iii) If D requests, judge must instruct jury not to infer guilt from silence (Carter) Sentencing judge is also not allowed to draw adverse inferences from silence at sentencing (Mitchell) Prosecutor may not comment on Ds silence (i) Question is whether the language used was manifestly intended or was of such character that the jury would naturally and necessarily take it to be a comment on the accuseds failure to testify (Dickinson) (a) Lockett: referring to evidence as un-refuted and un-contradicted does not violate the constitution because it does not add anything to jurys impression (b) Robinson: Where D urged that he was not allowed to tell his side of the story, prosecutions response that he could have taken the stand was ok; not used as substantive evidence, only to respond to Ds claim D has a constitutional right TO testify (Rock) (i) One of the few areas where adverse is an affirmative right (ii) Based on due process, 6th Amendment right to compulsory process, corollary to guarantee against compelled testimony (iii) State may place limits that are not arbitrary or disproportionate State may argue against Ds credibility by pointing out that he was present for all other testimony before taking the stand (Portuondo)

70

X.

Sentencing A. Introduction to Theories 1. Competing ideals: individualization v. equity 2. Rehabilitation a. Concerns with disparity and arbitrary/injustice if rehabilitation is unsuccessful b. Swing back towards rehab as a cost-cutting measure 3. Deterrence 4. Incapacitation 5. Retribution B. Competing Goals 1. Calibrate sentences to individuals culpability, dangerousness and need for deterrence (more discretion) 2. Equal treatment, avoiding unwarranted disparity (less discretion) C. Who determine type/amount of punishment for Ds conduct? 1. Legislature a. Set sentencing range for each offence b. Whether parole release available and when c. Good time amounts and rates d. What non-incarceration alternatives are available and when 2. Sentencing Commissions a. Set narrower sentence ranges or provide guidance to judges assigning initial sentences 3. Law enforcement a. Select sentence range by deciding which crimes to arrest/investigate (i) Including those carrying specific sentencing consequences 4. Prosecutors a. Exercising discretion (i) Initially and through pros, appeal and post-conviction b. Decide which crime to charge or dismiss (i) Including those with man mins c. Bargaining over sentence d. Recommending sentence 5. Judges (following juries in six states) a. Select sentence within range authorized for offense of conviction b. Reject/accept sentence terms negotiated by parties 6. Probation/Parole Officers a. Select sentence by deciding whether to seek revocation of probation or parole 7. Corrections Officials a. Deciding how much good time to allocate (i) Can be revoked b. Whether to release a person on parole 8. Governors/Board of Pardons a. Exercise discretion to commute sentence or grant pardon

71

D. Types of Sentences 1. Indeterminate a. Discretionary release on parole (i) i.e., must serve 5-10 years, at any point after 5 may be released by parole board b. Good time lower the time at which you are eligible for parole 2. Determinate a. Must serve full amount (i) i.e., sentenced to 10 years and must serve b. Good time lowers the full amount E. Constraints/Regulations on Sentencing Discretion 1. Prosecutors Discretion a. Judicial rejection of negotiated sentences or charge choice (i) Not common b. Political review (i) Elections c. Parole release where available d. Clemency 2. Of judges a. Statutory mandatory minimums (certain offense or if certain facts present) (i) Limits judges ability to depart downward thus it is up to prosecutor in selecting charges to determine b. Guidelines (which are now mandatory) (i) Base offense + offense characteristics downward departures / criminal history category c. Appellate review of compliance with mandatory minimums, guidelines, procedural requirements d. Political review (i) Judicial elections e. Parole release where available f. Clemency F. General Process 1. Jude receives information about offense and offender from proof or submission during trial or plea colloquy a. Often also from presentence report prepared by court staff (probation officer) after plea or trial 2. Hearing held where D (and victim) may speak, any contested points of law and fact are resolved and judge announces sentence 3. Subject to waiver by D

72

G. Const. Limits on Sentencing Procedure 1. Limits on Sentencing Info a. Judge can consider almost anything (any fact relevant to rehabilitation, remorse, etc. even if looks like an exercise of const. rights) (i) Information outside the record (Williams) (a) But in capital cases, must disclose which portion of the PSR formed the basis for the death sentence (Garner) (ii) Hearsay (Williams) (iii) Other crimes and conduct for which D was not charged or convicted (Williams) (a) Even if D was acquitted, standard is BRD and judge need only find by preponderance (b) Goes towards culpability and likelihood of rehabilitation (iv) Exercise of Rights (1st and 5th Amendment) (v) Ds commission of perjury (Dunnigan) (a) But the judge must make an independent finding to establish willful perjury, not just confusion, mistake, or faulty memory (b) Goes towards Ds character, respect for the law (c) May not increase above statutory max (vi) Cooperation and failure to cooperate (a) Can rely on non-cooperation in refusing to name drug supplier, but cannot use refusal to cooperate as a direct inference that Ps proof was correct because D has 5th A right to silence (Mitchell) (vii) Victims views (a) But state may give further guidance to court (Payne) b. But he may not (i) Retaliate for exercise of procedural rights (Pearce v. Blackledge) (ii) Evidence of Ds membership in Aryan Brotherhood was not relevant in killing where victim was white as well (violates 1st Amendment) (Dawson) (a) But a Ds anti-Semitic statements during trial were properly considered in arson of Jewish community center(Kapadia) (iii) Ds or Vs race or gender (McKlesky) (a) However, court has upheld hate crime statute that permits additional penalties for crimes committed with the intent to intimidate (animus towards a group is more culpable) H. Rights at Sentencing 1. Right to be present 2. Right to assistance of counsel (Mempa v. Rhay) 3. Right to present evidence a. Typically up to judges discretion b. If allocution, D must be placed under oath in some states c. States split on whether D has right to allocution 4. No right to discovery, disclosure or notice for sentencing factors 5. No right to explanation of reasons for sentence 6. No right to confront witnesses
73

I. Burden of Proof for Sentencing Factors 1. Element needed for conviction a. P has burden of proving beyond reasonable doubt on every fact necessary to Statutory Max: Jury constitute crime charged (McMillian) must find any fact that 2. Affirmative defense at trial raises statutory max a. D has burden by preponderance (Apprendi) or permits 3. Facts that lead judge to impose a higher sentence within the sentencing range punishment beyond what conviction alone a. P has burden by preponderance (Williams) allows (Blakely) (i) Judge has wide discretion to choose sentence and judge can pick any sentence within statutory range - Except criminal history (ii) There are facts relevant to sentencing that are not relevant to guilt (Almendarez-Torres) (a) Impractical to haul all PSR witnesses into court Statutory Range: 4. Fact of prior conviction that raises sentence max Judge has discretion a. P has burden by preponderance (Almendarez-Torres) within any range (i) Criminal history category available after 5. Fact mandating higher minimum conviction alone a. P by preponderance (McMillan/Harris) (Wiliams) (i) Merely a sentencing factor, not an additional charge that substantively Statutory Minimum: changed the indictment May be raised if fact(ii) Unless the fact that changes the nature of the conviction (wags the finding (McMillan) dogs tail?) 6. Fact raising statutory max a. Must be proven to a JURY beyond a reasonable doubt (Apprendi) (i) Within judges discretion to mandate a sentence WITHIN the range prescribed by statute, not to CHANGE charged crime (ii) Unless D ADMITS as part of conviction (Almendarez-Torres) (a) No issue about jury right to trial or burden of proof since not contested issue of fact (iii) Viewed as element of a greater offense: must also be charged in indictment (iv) Hate crime statute that boosts max sentence upon finding that D intentionally chose V based on religion, sex, race, sexual orientation (v) Unresolved issues: (a) Amount of loss each finding of loss doesnt raise the max (b) What counts as an admission? 7. Fact raising max sentence that would be authorized by conviction alone a. P must prove to JURY beyond a reasonable double (Blakely; Ring; Booker I) (i) If factfinding at sentencing exposes D to sentence greater than the statute authorizes through the guidelines, fact must be found by a jury (Blakely) (ii) If fact must be found (aggravators) to make D eligible for capital punishment, jury must find (Ring) (iii) Problems with Blakely (a) This holding threatens all discretionary sentencing guidelines: judges could no longer make calibrated findings of fact at

74

sentencing and judges would either have to be given broad discretion or guidelines would have to be advisory Federal system and most states responding by making guidelines advisory (Booker II) but judge must still stay within statutory range Some states limited the number of facts that put D in a different sentencing range and makes prosecution prove these facts at sentencing (b) Forces D to contest these factors during trial for guilt or innocence (iv) Note that this holding exempts findings of prior convictions in order to preserve the criminal history category in sentencing guidelines (a) But Besser (NY) found that persistent offender statute in NY violates these cases 8. A fact permitting consecutive sentences for multiple offenses a. P must prove by preponderance (ICE) J. Burden of Proof Sentencing Fact or Element 1. On Defendant a. Affirmative Defense 2. On Prosecution a. Preponderance (i) Fact used in setting sentence within indeterminate range (Williams) (ii) Fact of prior conviction that raises sentence maximum (AlmendarezTorres) (a) Has to have already been proven BRD or found by jury, thus certain of reliability (b) Failure to challenge by (iii) Fact mandating a higher minimum sentence (McMillan/Harris) (a) Does not adjust the max, merely the lowest amount served (iv) Fact required to run sentences for multiple convictions consecutively (Ice) b. Beyond Reasonable Doubt (i) Element (ii) Fact raising statutory maximum (Apprendi) (iii) Fact raising maximum authorized by conviction (Blakely/Ring/Booker) (iv) Fact requiring for fine amount (S. Union) (a) Questions arisen about McMillan/Harris as well as restitution and forfeiture

75

XI.

Double Jeopardy A. Purposes 1. Prevents the conviction of innocent through multiple rehearsals 2. Prevents government from harassing defendants by promoting finality 3. Prevents disproportionate punishment for given conduct B. Same Offense- Blockburger or Elements Test 1. Each offense must contain an element not contained in the other, otherwise, there is a presumption that the two offense are the same offense and cannot both be prosecuted or punished 2. Presumption can be rebutted by clear statement of legislative intent to permit separate punishment for both C. Collateral Estoppel (AsheI) 1. Issue preclusion bars prosecution for a different offense if: a. A FACT (element) necessary for conviction in the second prosecution b. Was NECESSARILY DETERMINED earlier in defendants favor c. By an ACQUITTAL in an earlier case D. Attachment 1. Bench trial a. Attaches when evidence is received and the governments case begins (document or first witness sworn 2. Jury Trial a. When jury is sworn (Crist) (i) Jeopardy is an interest in avoiding harassment and embarrassment; having evidence presented to two juries creates double jeopardy (ii) Ds interest in retaining the chosen jury through verdict; bound up in the idea that jury acquittal is special (iii) No DJ bar if dismissed pre-trial or after voir dire 3. Guilty Plea a. Once the plea is accepted by the judge E. Reprosecution Following Mistrial 1. Where D did NOT consent or request mistrial a. DJ bars unless there was manifest necessity in granting mistrial (i) Manifest necessity exists if an impartial verdict cannot be reached or if reversal is certain (incurable erroros) (a) Examples Indictment fails to state a crime (Sommerville) Jury deadlock (Perez): permissible to declare a mistrial rather than force continued deliberations depending on whether D objected, length of the trial, length of deliberation, effort to encourage/break deadlock (Allen instruction, dynamite charges) Jury misconduct (juror shows video to other jurors of D confessing on iPhone) Defense counsels outrageous opening statement (Arizona v. Washington) (ii) No manifest necessity where:
76

1. Has jeopardy attached? 2. What was the disposition?

(a) Alternatives excised that would have allowed trial to continue Substitute juror or proceed with 11 (for drunk/biased a.i. Mistrial on Ds juror) motion no DJ Give instruction (D/witness outburst) a.ii. Mistrial on P: DJ Grant continuance (judge or D attorney sick) unless manifest necessity Make jury deliberate longer b. Acquittal DJ bar (b) Delay is due to foreseeable P error or P manipulation Witness missing; not ready (Dwonum) 2. If D CONSENTED or requests mistrial a. Considered to have waived DJ protection (Dinitz) c. Conviction overturned (i) UNLESS D was goaded by P into moving for a mistrial (Oregon v. on appeal i. Evidence DJ bar Kennedy) ii. Procedural no bar (a) Goading is present if the record supports an inference that P thinks trial is going badly and tried to manipulate retrial and DJ d.i. Dismissal on system evidence, insanity, (b) Depends on Ps intent entrapment DJ bar d.ii. Dismissal on Must deliberately provoke D into moving for mistrial procedure no bar (not just do something stupid) (ii) Minority of states offer more protection 3. If acquittal or standing (a) Retrial also barred if mistrial is in response to egregious conviction, cannot bring new prosecution for prosecutorial misconduct regardless of prosecutors hope to same offense abort trial (b) Californias subset of prosecutorial conduct that will not bar retrial is limited to intent F. Prosecution Following Acquittal 1. Double jeopardy attaches after an acquittal (failure of/insufficient proof) a. State cant appeal as it would amount to a retrial (i) Goal is to prohibit affording the prosecution another opportunity to supply evidence which it failed to muster the first time around (DiFrancesco) b. Protects the potential for jury nullification (Weston) 2. What counts as an acquittal a. Jury verdict of not guilty b. Bench trial (judicial) verdict c. Directed verdict before jury verdict d. Dismissal (at trial or on appeal) for insufficient evidence e. Dismissal based on failure to rebut insanity or entrapment defenses f. Where D convicts of lesser-included, imply acquittal of greater charge (Green) (i) Unless hung jury or jury specifically notes it cannot reach agreement on higher charge 3. Unless D bribed the judge or jury (People v. Aleman, Ill.)

77

G. Reprosecution Following Dismissal 1. Pre-Trial dismissal does not bar reprosecution because DJ hasnt attached (Serfass) 2. Appeal allowed when aborted in favor of D during trial for reasons unrelated to sufficient of proof (Ball) a. Insufficiency of the information/charging (Lee) b. Pre-indictment delay does not bar retrial because it is actually procedural (Scott) (i) P could appeal judges ruling and obtain retrial c. If statute of limitations has run, state can bring a new charge that includes a more recent overt act (Kruelski) d. Improper venue: more procedural than substantive (Wilkett) 3. Dismissal based on failure to rebut insanity or entrapment defense bars reprosecution (Burks; Scott) a. Such a finding establishes the Ds lack of criminal culpability (Russell) 4. A dismissal for insufficiency of the evidence bars subsequent prosecution a. When judges ruling represents a resolution in Ds favor on some or all of the factual elements of the offense charged (Sanabria; Scott) (i) Even if the judges ruling to exclude evidence that leads to declaration of acquittal is clearly erroneous, counts as an acquittal (Sanabria). (ii) Even if judge dismisses because he erroneously believed there was an extra element in the charge, counts as an acquittal 5. Dismissal following a jurys guilty verdict a. No DJ bar (Wilson) (i) If P successfully appeals the dismissal, then the verdict is simply reinstated; D would not be subjected to second trial H. Reprosecution Following Conviction Overturned on Appeal 1. If D appeals convictions and it is (ultimately) reversed for insufficient evidence of one of the elements a. Double jeopardy bars retrial (Burks) (i) Guilty Verdict + JNOV Govt can appeal (a) If AFFIRMED, cannot retry (ii) Guilty verdict + Reversed on Appeal for Insufficient Cannot Retry (iii) Guilty verdict + Court of appeals affirms conviction D can seek Supreme Court review of sufficiency (a) IF S.Ct. finds insufficient evidence, cannot retry 2. If D appeals and conviction is reversed for procedural error (anything other than insufficiency of the evidence) a. Double jeopardy does not bar retrial (Ball) (i) No matter how flagrant the prosecutors error 3. If court of appeals finds that trial judge shouldnt have let the evidence in and without it there is insufficient evidence a. DOES NOT bar retrial because it is a procedural error (admitting evidence) (Lockhart) 4. Where D is convicted of lesser offense without jury statement regarding greater a. Implies acquittal of greater, thus DJ (Green)
78

b. Retrial on lesser is allowed (Ball) c. If prosecution retries both and trial court erroneously denies Ds double jeopardy objection on charge from which implicitly acquitted (i) If retrial resulted in conviction only on the charge that was not jeopardy-barred, new trial (Price v. Georgia) (a) Cannot determine whether or not the higher charge (implicitly acquitted jeopardy-barred) induced jury to find guilty of less erroneous offense rather than continue to debate innocence (ii) If retrial resulted in conviction for higher (jeopardy-barred) charge, Court reduced level of conviction to lesser included (Morris v. Matthews) (a) By finding guilt on higher charge jury had necessarily found, without suggestion of possible compromise, that state established all elements of lesser included 5. Where D is convicted of lesser but hangs on greater a. Retrial on greater is not barred (Sattazahn) I. Judicial Acceptance of Guilty Plea to Lesser 1. No inferential findings of not guilty as to higher charge, thus not barred by Green a. BUT if pros. agreed to dismiss in return for plea, D may enforce that agreement J. Following Sentencing 1. Where judge misconstrued sentencing law and government can appeal, no DJ (DiFrancesco) 2. Double Jeopardy APPLIES to one type of sentencing decision a. Capital Sentencing (i) Trial type hearing focusing on special factual standards governing the imposition of the death penalty (ii) Jury determination not to impose comparable to trial and acquittal as to that setnece K. After Acquittal or Standing Conviction 1. Double Jeopardy: Same offenses must be joined or DJ will bar prosecution for the 2nd offense a. Does each offense have on separate element not contained in the other? (Dixon-Blockburger) If one offense is a lesser-included, barred by double jeopardy from bringing separate prosecutions (i) Dixon: Contempt + drug crime = same offense; contempt (for drug crime) and assault with intent to kill = not the same offense, intent to kill is an extra element 2. Collateral Estoppel/Issue Preclusion a. Distinguish from DJ: different victim means its not the same offense; look to claim preclusion

79

b. Acquittal for one offense will bar a later prosecution for a different offense if a fact necessary for conviction of the second offense was necessarily determined earlier in the Ds favor (Ashe) (i) Was the fact fully-litigated? (ii) Look at charges to the jury, arguments to the jury, questions by jurors (iii) If D conceded all other issues, acquittal was probably based on that issue c. After losing civil case, cannot seek criminal prosecution if (a) actually litigated and (b) fact was reason for finding at earlier trial (i) But if criminal prosecution fails, government can seek civil penalties because the burden of proof is different L. Where Multiple Attempts to Convict ALLOWED 1. When legislature punished same conduct 2. When prosecution ends before jeopardy attached 3. After mistrial when D consents 4. After mistrial when D objects, if there is manifest necessity 5. When dismissed after jeopardy a. Not acquittal (Scott, Lee) 6. When D establishes procedural error (Ball) 7. By separate sovereigns (different states state/fed) a. Sovereign specific right thus can be tried in separate courts 8. After assault conviction, if victim dies, can be prosecuted for homicide (Diaz)

80

XII.

Review of Error by Appeal A. Source of Defendants Right to Appeal 1. No constitutional right to appeal (Halbert v. Michigan) 2. But if a state provides a right to appeal, the Constitution requires state to comply with DP and EP a. DP (i) No vindictive prosecution in response to exercising right and getting relief on appeal (Pearce) b. E (i) Must provide counsel for indigents on direct appeal (Douglas) c. EP (i) Cannot deny appeal to indigent D (Griffin) B. Defendants Appeals 1. Can D appeal at this time? a. Final Judgment Rule (i) D cannot appeal until sentence is imposed (a) More efficient to resolve all at once, appeal may crate undue delay and lead to lost evidence; one of the painful obligations of citizenship is that you have to wait until final judgment to appeal (Cobbledik) b. Unless there is an exception for a collateral issue (Coopers & Lybrand) (i) Order conclusively determines the question (not tentative, informal, or incomplete) (ii) Issue is important (iii) Issues is independent of the merits (a) Wouldnt be affected by subsequent decision on the merits (iv) Normal appellate review would provide inadequate remedy (a) Review would not cure error (b) Harm would have already occurred c. Rulings on which D can seek interlocutory appeal (i) Denial of bail (Stack v. Boyle) (a) Finally resolved and independent of the issue to be tried; moot if review awaits conviction (ii) Conditions of pretrial release (iii) Double jeopardy (a) Constitutional right is the right not to be retried for the offense (Abney) (iv) Speech and Debate clause (a) Right not to be questioned or tried about certain legislative activities (Helstoski) (v) Motion for return of property (vi) Order requiring juvenile to be tried as an adult (a) Right would be lost (vii) Sell hearing forcing D to be medicated (viii) Third party rights/ Media gag order (a) Affects 1st Amendment rights of access
81

d. May not seek interlocutory appeal for (i) Vindictive prosecution (a) Remedy is a dismissal, not a right not to be tried (Hollywood Motor Cars) (ii) Speedy trial violations (a) Determination of whether a violation has occurred depends on a determination of prejudice at trial (MacDonald) (iii) Grand jury errors (a) Appeal involves considerations enmeshed in merits of the case, even despite Mechanik rule that violations are per se harmless (iv) Qualification of counsel (a) Tied up with merits of trial and prejudice (v) Improper joinder (vi) Improper venue (a) remedy is transfer back to proper venue (vii) Denial of MTD due to statute of limitations (a) not as absolute as DJ (viii) Denying Ds demand for discovery (ix) Requiring D to produce discovery (a) Unless Ds counsel refuses and is placed in contempt (x) Courts are split regarding MTD based on violation of immunity agreement (a) tied up with merits if there is breach, but it is largely independent, unappealable because D is already harmed by trial 2. Did D expressly waive his right to seek relief based on this error? a. No relief unless error in waiver or jurisdictional (i) Plea agreements (ii) Stipulation (iii) On-the-record waive 3. Did D forfeit right by pleading guilty? a. Generally waive all pre-plea errors (i) Except (a) Unless error in plea itself Ineffective, involuntary waiver (b) Error that cant be cured/bar second try Vindictive charging, GJ selection challenges (c) Error expressly reserved by conditional plea agreement AND IAC and DJ violations that are clear on the fact of the charge 4. Did D forfeit right by failure to raise before the trial court in a timely appropriate manner? a. Why bar? (i) Advance most efficient and fair resolution of claim (time specific) (a) Could have been taken care of, cured, or resulted in dismissal at that point if raised (ii) Prevents unicorn (Henderson)

82

(a) Those who believe that defense attorneys decide deliberately to not raise in hopes of getting appellate relief b. Unless plain error- FRCrP 52(b) [Olano; Johnson; Dominquez Benitez; Puckett; Henderson (2013)] (i) Error (a) Not expressly waived or waived by statute (STA) (ii) Plain (a) Obvious by time of appeal (iii) Affected substantial rights (a) D has burden of showing that it made a difference in the outcome (iv) Seriously affects the fairness, integrity, or public reputation of judicial proceedings: bad enough 5. Was there error and can it be established under the relevant standard of review? a. Abuse of discretion where judge had discretion (i) Evidentiary (ii) Sanctions for discovery abuse (iii) Challenges for cause b. Clearly erroneous: findings of fact (i) Whether prosecutor intended to use race as basis for peremptory (Batson) (ii) Whether client was advised on max sentence before plea c. De novo: Mixed questions of law and fact (i) Strickland: whether counsel was reasonable (ii) Dangerousness at bail hearing (iii) Manifest necessity for mistrial 6. Assuming error not waived or forfeited, relief? a. Double jeopardy, vindictive charging, and Speedy Trial constitutional error: remedy is barring reprosecution b. Where D must show prejudice to establish error itself, no need for harmless error analysis: (i) Brady: requires showing of reasonable probability that failure to disclose affected the outcome (ii) Darden: Improper argument (iii) Strickland: Ineffective assistance of counsel (iv) Youngblood: Improper destruction of evidence c. Structural errors: Impacts framework of trial and affects basic protections, without which, trial is fundamentally unfair; automatic reversals (Fulminante; Rose) (i) Improper exclusion of capital punishment juror because of views on DP (Gray) (ii) Racial discrimination in selection of grand jurors (Rose) (iii) Batson errors (iv) Denial of public trial (Waller) (v) Defective BRD instruction (Sullivan) (vi) Denial of self-representation (McKaskle)
83

(vii) Biased judge (Tumey v. Ohio) (viii) Denial of counsel (Gideon) (ix) Improper withdrawal of appointed counsel (Penson) (x) Counsel of ones choice (Gonzales-Lopez (xi) Conflicts where constitution mandates an inquiry (Holloway) (xii) Denial of right to consult with counsel during overnight recess (Geders) d. Constitutional errors: P must show beyond a reasonable doubt that the constitutional error was harmless (Chapman) (i) Admission of co-D statement in violation of confrontation could be harmless (ii) Griffin violation could be harmless (iii) Introduction of coerced confession could be harmless (Fulminante) (iv) Denial of counsel at PH can be harmless (Coleman) (v) Improper instruction on one element of the offense violates Sixth Amendment jury trial guarantee, but can be harmless (Neder) (a) Where the government does not produce evidence to dispute the element = harmless error e. Statutory errors are subject to harmless error analysis: Government must show error had no substantial effect on the verdict (Substantial effect is lower than BRD) C. Governments Right to Appeal 1. No constitutional right to appeal 2. Federal system and most states allow prosecution appeal a. Criminal Appeals Act of 1970: May appeal from decision, judgment, or order dismissing indictment or information or granting new trial after verdict or judgment except where appeal would violate Double Jeopardy clause 3. Prosecutor may have more opportunities for interlocutory appeal because this is their only chance a. Examples of interlocutory (i) Motion to suppress: if USAO certifies that appeal is not taken for purpose of delay and exclusion relates to a substantial fact of the case, P may appeal pre-trial (ii) Midtrial order directing government witness to respond to defense questions on matter claimed to be privileged: witness would have to refuse and court would hold witness in contempt so witness can appeal contempt order b. Rulings relating to sentences: P can always appeal sentences and rulings related to sentences (dont have to do with acquittal) so no need for special right to appeal

84

You might also like